Vajad kellegagi rääkida?
Küsi julgelt abi LasteAbi
Logi sisse

Kriminoloogia konspekt (1)

5 VÄGA HEA
Punktid

Esitatud küsimused

  • Millest õpik rääkis?
  • Milline on antud lähenemisviisi suhtumine inimesse?
  • Kust tulevad normid mis reguleerivad käitumist?
  • Mida kujutavad endast hälbed nendest normidest?
  • Mida antud lähenemisviis peab hälvete põhjusteks?
  • Kuidas suhtub antud lähenemisviis hälvikusse?
  • Millised on selliste vahendite kasutamise eesmärgid?
  • Kuskil on üldse vaba tahet?
  • Kuidas füüsilised faktorid mõjutavad kuritegevust?
  • Kuidas RTT jõutakse?
  • Kuidas märgid mõjutavad inimest?
  • Kuidas indiviidid end ise märgistavad?
  • Millise konkreetse teoga märk kinnistatakse?
  • Miks otsida põhjuseid ühiskonna tasemel?
  • Millest ta sõltub?
  • Miks anoomia tekib muutuste ajal?
  • Mis juhtuks kui õnnestuks kuritegevus likvideerida?
  • Kuidas latentseid kuritegusid uurida?
  • Mis siis ühiskonnas toimuks?
  • Mida situatsioonivahenditega püütakse saavutada?
  • Kes peaks kinni maksma naabrivalve projekte?

Lõik failist

KRIMINOLOOGIA TÄISKONSPEKT
Kriminoloogia koht teadusharude süsteemis. Kriminoloogiliste teooriate süstematiseerimine.
Kriminoloogia põhimõisted
Esituskeel – märkide, sümbolite, mõistete süsteem, mille abil ja mille kaudu esitatakse antud teaduse väited, tõestused ja järeldused. On levinud arusaam, et keele matematiseerituse aste väljendab seda, kas tegu on teadusega või mitte. Ühiskonnateadused aga uurivad kvalitatiivselt, mitte kvantitatiivselt.
Uurimisobjekt on inimene tema ühiskondliku olemise eri vormides ja ilmingutes, seega peab olema ka keel mitmemõõtmeline, nt sõnas „ kuritegu “ põimub vähemalt kolm dimensiooni : materiaalne (tegu kui nähtus), aksioloogiline (teatavast väärtusest lähtuv hinnang teole ), formaal-juriidiline (tegu kui karistatav ). Kriminoloogias on teaduskeel siiski alles algusfaasis
Kriminoloogia kui teaduse seosed teiste teadustega (kriminaalõigusteadus, sotsiaalteadused , hingeteadused)
Kuriteo ja kurjategijate vastu tunnevad huvi eelkõige kriminaalteadused, teisalt pakub see ja muu sotsiaalne patoloogia ka sotsiaalteadustele.
Kriminaalteaduste seos kriminoloogiaga: tegemist on teatavas mõttes üksiku ja üldise vahekorraga. Iga üksik kriminaalteadus jõuab oma tunnetustegevuses ja arengus varem või hiljem arusaamisele, et midagi olulist jääb tema haardeulatusest väljapoole. Samal ajal mõistetakse, et nimelt see on oluline. Hõlmamaks konkreetsete kriminaalteaduste uurimisorbiidist välja jäävat, kuid kuriteo ning kuritegevuse adekvaatse tunnetamise seisukohalt tähtsat nähtuse- ning probleemiringi, ongi tekkinud ja arenenud kriminoloogia. Ta on metateadus kriminaalteaduste jaoks. Ta uurib probleeme, mis on iga üksiku kriminaalteaduse aspektist fundamentaalsed.
Sotsiaalteadusena on kriminoloogia tihedalt seotud teiste sotsiaalteadustega nii uurimisvaldkonna kui meetodite kaudu (võtab üle saavutusi ja pakub teistele väärtuslikke andmestikku uuritavate sotsiaalsete süsteemide anomaalsete seisundite kohta). Sarnane on seos ka hingeteadustega.
Kriminoloogia klassikalised koolkonnad normativism ja positivism
Normativistlik paradigma – juured 17. sajand (Grotius ja loomuliku õiguse põhimõtted)  loomuliku õiguse kontseptsioon ongi kajastatud kriminoloogia normativistlikus paradigmas, kusjuures idealistlikult meelestatud kriminoloogid käsitavad nn loomuliku kriminaalõiguse allikana inimese olemust või siis inimesest kõrgemal seisvat alget. Nad usuvad, et seadused on loomuliku kriminaalõiguse projektsioon.
Normativistliku kriminoloogia teene : kurjategija isiksuse problemaatika esiletõstmine. See sunnib ka pidevalt tegelema kriminaalseaduse kvalitatiivse parandamisega sotsiaalse õigluse saavutamise nimel.
Kuivõrd kriminaalseadus tuleneb inimese või tema ühiskondliku olemise olemusest, teenib ta objektiivselt inimese huvisid. Järelikult need, kes rikuvad kriminaalseadust, toimivad oma huvide vastu  ebanormaalsus tingib kuriteo toimepanemise. Tugev külg on eetiline suunitlus .
Positivistlik ( sotsioloogiline ) paradigma – 19 saj II pool – eelistab seaduspärasusi ja fakte. Lähenetakse läbi ühiskonna, väliste asjaolude.
Selle teene: püüab arendada faktoloogiale rajatud teooriat, laiendab kriminoloogia tunnetuslikke võimalusi. On saanud selgeks, et kuritegevuse olemuslikke momente pole võimalik mõista lahus ühiskonna elutegevuse teiste külgede tundmaõppimisest. Kriminaal - ja karistuspoliitiliste programmide asemel pakutakse rohkem sotsiaalmajanduslikke ja kultuurilisi preventsioonimeetmeid.
Need paradigmad täiustavad teineteist.
Kriminoloogiateadus Eestis
1920-1940 – episoodiliselt, ülikoolis polnud. Prof . Kadari arvas selle abiteaduse hulka. Ta peab kriminoloogia osadeks kriminaalpsühholoogiat, kriminaalantropoloogiat, kriminaalsotsioloogiat, kriminaalbioloogiat.
Politseileht kajastas tuntumaid teooriaid , statistikat.
1920-1930 – uuriti alaealisi kurjategijaid ja nende sugulasi.
1960 – sündis teadus ja õppedistsipliin TÜ-s (see oli normativistliku põhikoeta empiiriline teadus lähtuvalt NL-st)
1970 II pool – murrang sotsioloogilise lähenemisviisi kasuks, välja anti TÜ Toimetiste vihikut „Kriminoloogia- alaseid töid“.
  • Kuritegevusandmete võrdlemine ajas ja ruumis
    Muutusi registreeritud kuritegude arvus võivad põhjustada üks või mitu järgmistest asjaoludest:
    • tegelikud muutused kuritegevuses
    • muutused aktiivsuses
    • muutuesd seadusandluses, kuritegude registreerimise korras
    • muutused elanikkonna valmisolekus kuritegudest teatada

    Tuleb arvestada võrdlusandmeid, mille kohta midagi näidata, demograafiat, juhuslikkust. Varavastaste kuritegude puhul on kuritegude arv suurem ja tendentsid kergemini määratletavad. Varavastased kuriteod moodustavad ka enamuse.
  • Kuritegevusandmete geograafilis-territoriaalne analüüs
    Huvi pakub regionaalne levikupilt, taustategurite omavahelised seosed. Peab arvestama erinevat inimeste valmisolekut kuritegudest teatada piirkonniti, ka arvestada politsei töökoormust regiooniti.
  • Eri sotsiaalsete gruppide kuritegelik aktiivsus
    Huvi paku eri gruppide osakaal, kuritegelik aktiivsus (gruppide osakaal kurjategijate vs kahtlustatute suhtes). Eestis: eelkõige muulasest mehed vanuses 16-24. Need näitajad sõltuvad aga tugevalt hoiakust kuritegevuse suhte, politsei tööst. Vähe lahendatakse ju vargusi ja valgekraede kuritegusid .
    Millest õpik rääkis?
    Kuritegevuse teooriate eesmärgid:
  • aitavad süstematiseerida olemasolevat informatsiooni kuritegevuse kohta ja pakuvad lähtekohti uuteks uuringuteks
  • võivad olla kuritegevuse kontrolli, vähendamise või ärahoidmise programmidele teoreetiliseks aluseks
    Teooriate rühmad:
  • bioloogilised
  • ühiskondlikud
  • hüpoteetilised sisemised psüühilised struktuurid
  • indiviidi kogemuse omandamine ja isiksuse kujunemise protsessiga seotud
    Emile Durkheim – üks esimesi, kes püüdis selgitada, et kuritegevus on ühiskonnas normaalne nähtus. Kuritegevuse likvideerimiseks on vaja kõik inimesed suruda täielikult ühtsete käitumisreeglite alla. See aga teeb arengu võimatuks, sest igasuguse sotsiaalse arengu eeltingimus on muutused inimeste käitumises, mis on ühesugususe korral võimatu.
    Rober K Merton – ta näitab reljeefselt, kuidas ühiskond ise, püstitades oma liikmete ette eesmärke, kuid jättes välja nende eesmärkide saavutamiseks vajalike vahendite seaduslikkuse , suunab oma liikmeid otsima ja kasutama ka ebaseaduslikke vahendeid.
    Thorsten Sellin – kultuurikonflikti reooria – erinevate kultuuride kokkupõrked võivad põhjustada kuritegelikku käitumist. Igal kultuuril on omad käitumisnormid ning osa tegusid , mis ühtede normide puhul on lubatud, on teiste järgi keelatud ja isegi kuritegelikud. Ning kui isik, kes on omandanud ühed normid, satub teiste normide mõjupiirkonda, siis võib ta toime panna kuriteo.
    Sykes ja Matza – neutraliseerimine – näitab, kuidas toimub kurjategija sisemine psüühiline kaitse. Kurjategijad omandavad neutraliseerimise meetodid, mille abil nad püüavad endale ja teistele seletada, et nemad ei ole toimepandud kuriteos süüdi või see polegi kuritegu.
    Kuritegevuse seisund Eestis
    Andmetesse tuleb suhtuda kriitiliselt – kõiki ei registreerita (50:50; hirm ühiskonna vastukaja ees või ajakulu või lootusetus või abstraktne kannatanu).
    Kui võrrelda praegust ja nõukogudeaegset kuritegevust. Registreeritud kuritegevuses on umbes kaheksakordne vahe. Registratsioon on küll parem, aga osa kuriteoliikide puhul on kasvu siiski olnud. Kaks kiiret kasvuperioodi: 1989-1992 (ühiskondlikud muutused, kõigis postsotsialistlikes maades), 1997-2000 (suur vahe inimeste püstitatud eesmärkide ja tegeliku olukorra ja legaalsete vahendite vahel – Merton  börsikrahh, majanduslik langus). Kui võrrelda nõukogude ajaga , on vahe umbes kaheksakordne (nüüd on kergem toime panna vargust).
    1980 aastal registreeriti Eestis samuti kuritegevuse oluline kasv. Seda on seletatud ettevalmistustega olümpiamängude toimumiseks. Kui 1984 aastal oli sotsialistlik kord kõige madalamas seisus, siis oli samuti märgata mõningast kuritegevuse kasvu. 1985 - 1986 aastal toimunud kuritegevuse taseme langust on seletatud alkoholivastase võitlusega (nn. kuiv seadus), kuid samasse perioodi langeb, tänu alkoholi müügi piiramisele, toksikomaania tekkimine. Seega näeme, et vähemalt üks osa kuritegevusest on väga kiire dünaamikaga ja seetõttu tuleb seletusi otsida kiiresti muutunud sotsiaalsetest tingimustest. See ei välista siiski aga kuritegevuse suhteliselt stabiilsema osa võimalikku mõningast tingitust kurjategija isikuga seonduvatest faktoritest, sh. ka bioloogilistest faktoritest.
    NB! Paljud teod jäävad registreerimata (ei ole konkreetset kannatanut või ei viitsita). Suuti muudatusi kuritegude arvus võib põhjustada nt muudatus registreerimise süsteemis. Palju seksuaalkuritegusid jäi varem teatamata, sest kardeti negatiivset vastukaja ühiskonnast. Alates 2007 dekriminaliseeriti pisivargused.
    Kuritegude registreerimine vs avastamine:
    1989-1992 perioodil kuritegevus hüppeliselt kasvas, kuid avastamine vähenes (ressursipuudus, oskuste puudus). Siin tekkis oht, et kuritegevus hakkab väljuma kontrolli alt. Siiski 1992. aastal olukord paranes . Kohtusse laekumine on veel oluliselt väiksem.
    Protsentuaalselt on kunagi olnud nad kunagi 100% läheduses (nõukogude aeg; sõltub registratsioonist). Avastuse protsent langes kuni 1992. Praegu see kasvab.
    Kuritegude arv 10 000 elaniku kohta:
    Juhib Harju maakond 492 inimesega, Ida-virumaa teisel kohal 329-ga. Edasi tulevad Tartu 319, Pärnu 314, Valga 295, Lääne 290, Rapla 264.
    Levinumad kuriteod 2005-2006:
    Vargus (u 26000 / 30 000), joobes juhtimine (u 4150 / 3500 ), kehaline väärkohtlemine (3700 / 3456), kelmus (1968 / 2127), avaliku korra raske rikkumine (1500).
    Kuritegevuse dünaamika võimalik seletamine erinevate kriminoloogiliste teooriatega
  • Kuritegevuse seletamine läbi isiku – seosed tunnuste ja tegude vahel  mikro -tase.
    Ajalooliselt on võimalik vaadelda mitmeid erinevaid lähenemisviise kuritegevusele , milledes on püütud seletada kuritegevust läbi kurjategija isiku. Selleks, et anda nendest süstemaatilisem ülevaade, püüame grupeerida neid vastavalt sellele, milliseid vastuseid nad pakuvad alljärgnevatele küsimustele:
    1. milline on antud lähenemisviisi suhtumine inimesse ?
    2. kust tulevad normid, mis reguleerivad käitumist?
    3. mida kujutavad endast hälbed nendest normidest?
    4. mida antud lähenemisviis peab hälvete põhjusteks?
    5. kuidas suhtub antud lähenemisviis hälvikusse?
    6. millised vahendid pakub antud lähenemisviis välja hälvikute käitumise normaliseerimiseks?
    7. millised on selliste vahendite kasutamise eesmärgid?
    Ajalooliselt on vaadeldavatest lähenemisviisidest kõige vanem kristlik-religioosne lähenemine.
  • Seletamine läbi ühiskonna kui terviku – seosed ühiskonda kui tervikut iseloomustavate andmete ja kuritegevuse vahel  makro-tase.
    Ajalooline kristlik religioosne mudel:
  • Inimene on patu kandja.
  • Normid on religioossed käsud.
  • Hälbe olemuseks on patt.
  • Normi rikkumise põhjuseks on mõju väljastpoolt.
  • Hälvik on patune .
  • Inimene normaliseerub pattude lunastamisega läbi kannatuste.
  • Eesmärgiks on kahetsus ja hinge puhastamine.
    Sellist mudelit praegu keegi ükski kriminoloogiline teooria tõsimeeli välja ei paku, kuid samas ei tohi unustada, et see teooria oli Euroopas valitsev sajandeid . Kõige kujukamalt iseloomustab antud mudelit inkvisitsioon . Esimesed inkvisitsiooni ohvrid on teada 4. sajandist ja viimased 19. sajandist (Hispaanias).
    Vaba tahte mudel:
  • Inimene on vaba tahtega isik, kes püüab kogeda naudinguid ja vältida kannatusi.
  • Normid tulenevad ühiskondlikust lepingust.
  • Hälve on ühiskondliku lepingu rikkumine.
  • Rikkumine tuleneb inimese otsusest nautida teiste arvel.
  • Hälvikusse suhtutakse kui inimesse, kes oma eesmärkide nimel on otsustanud teiste õigusi rikkuda.
  • Inimene normaliseerub, kui tekitada neile kannatusi proportsioonis saadud naudingutega.
  • Eesmärgiks on süüdlases teiste huvidega arvestamise motiivi tugevdamine.
    Sellele teooriale tugineb avalik arvamus, et kuritegevust on võimalik vähendada ainult karistuse kandmisega.
    Naudingut saadakse 1 ühik, sellele peab vastama kannatus ( formaalne karistus , millele lisandub moraalne karistus). Kannatuste 1 ühik korrutatakse aga vahelejäämise tõenäosusega ja riskiprotsendiga ja karistuse lähedusega jne.
    Probleeme tekitab antud lähenemisviisi puhul küsimus, et mida mõista naudingu ja kannatuste proportsionaalsuse all. Esmapilgul tundub, et vahekord peaks olema 1:1, kuid sinna ette tuleb lisada veel kannatuste hulka suurendav koefitsient, mis on iga juhtumi puhul erinev st. 1:k*1. Koefitsient lisandub selle tõttu, et väga oluline osa kuritegusid ei satu üldse registratsiooni (meil umbes 50%). Registreeritud kuritegudest selgitab politsei välja umbes 1/3 ning seega umbes 17% kõigist kurjategijatest jõuab karistuseni. Koefitsient tekib kuna karistatakse oluliselt vähem kui on kuritegusid. Nii jõuame me esialgsest 1:1 proportsioonist üsna kaugele.
    Arvestada tuleks ka seda, et karistamine ei toimu kunagi vahetult kuriteo sooritamise järel. Ühiskonna reaktsioonid on hilinenud ning paljudel juhtudel ei reageerita üldse. Seetõttu võtab kurjategija potentsiaalset karistust arvesse märksa vähemtähtsana kui siis, kui karistus saabuks kohe vahetult kuriteo toimepanemise järel. Seetõttu tuleb lisada veel teinegi koefitsient, 1:k¹*k*1. See nõuaks aga juba karistuse muutumist vastuvõtmatult raskeks, mis pole objektiivsel ja ka subjektiivsel põhjustel võimalik.
    Siiski tuleb tunnistada, et vaba tahte mudel on kõige levinum. Suurele osale inimestele on omane mõtteviis, et kui kasutada võimalikult rasket karistust on võimalik kuritegevust oluliselt piirata.
    AGA:
    • Kas kuskil on üldse vaba tahet? – geneetiline, ainevahetuslik jne.
    • Teistega arvestamine (inimesed, oma järeltulijad) – oleme programmeeritud nii, et oskame tunda naudingut teistele meeldimisest .

    Valgustusfilosoofial põhinev mudel:
  • Inimene on vaba ja mõistlik – rõhk mõistlikkusel.
  • Normiks on mõistlik ja otstarbekohane käitumine.
  • Hälve on mõistliku ja loomuliku korra rikkumine.
  • Hälbe põhjuseks on teabe puudumine – ei tea, mis on mõistlik ja otstarbekohane käitumine.
  • Hälvikusse suhtutakse kui harimatusse ja ebateadlikku inimesesse.
  • Normaliseerimise vahendid on õpetamine ja teadmise levitamine.
  • Eesmärk on valgustatud ja haritud isiku kujundamine.
    Lähenemisviis pärineb valgustusajast, kuid oma mõju omab ta ka kaasajal (kuigi enam polda nii naiivsed). Selle sajandi 1960 - 1970 aastatel leidis antud teooria kõlapinda Nõukogude Liidus, kus leiti, et on vaja parandada õiguskasvatust, anda õigusteadmisi, et ära hoida õigusvastast käitumist. Ka kaasajal võib periooditi kohata mõttekäike, et kuritegevuse preventsiooniks on vaja parandada õigusalaste teadmiste levikut.
    See on ligilähedane ühiskondlikkule leppele. Praeguseks on see mudel suurema populaarsuse kaotanud. Seda on kontrollitud empiiriliste uuringutega ja tulemused näitasid, et tegelikkus selle mudeliga kokku ei läinud. Ginter kahtleb uuringute õigsuses.
    Moraalist lähtuv käitumise mudel:
  • Inimene on moraalne olend, kes lähtub kategooriatest hea ja kuri.
  • Normiks on ühiskonna moraalile vastav käitumine.
  • Hälve on amoraalne akt st kurja tekitamine.
  • Hälbe põhjuseks on kasvatusvigade või halva eeskuju tõttu tekkinud moraalidefektid.
  • Hälvik on amoraalne inimene, kes on kõlvatute vaadete ja ettekujutustega.
  • Normaliseerimise vahenditeks oleks inimese kasvatamine ja parandamine sh vaadete ja veendumuste muutmine.
  • Eesmärgiks on moraalsele, heale orienteeritud inimese kujundamine.
    Kas sellised vahendid on kättesaadavad, et inimeste vaateid ja veendumusi muuta. See on lihtsam ilmselt alaealiste puhul. Samuti on see võimalik vaid väga väikse rühma peal – neid inimesi, kes suudavad teisi nii mõjutada, on vähe.
    See mudel on kõgie rohkem omaks võetud.
    Bioloogilis-psühholoogiline mudel:
  • Inimene on eelkõige bioloogiline, psüühikaga mudel.
  • Norm on füüsiliselt ja psüühiliselt normaalse isiku käitumine.
  • Hälve on füüsiliselt või psüühiliselt defektse isiku käitumine.
  • Hälbe põhjuseks on kas kaasasündinud või omandatud füüsiline või psüühiline defekt .
  • Hälvik on füüsiliselt või psüühiliselt defektne isik.
  • Normaliseerimise vahendina pakutakse psüühilist või somaatilist ravi või kahjutukstegemist.
  • Eesmärk on kahjutu või terve isik.
    Siin on märgata rõhutatult ratsionalistlik lähenemine.
    Kahjutu isiku mõiste sissetoomise põhjuseks on asjaolu, et kui kuriteo toimepanemise põhjus on kas füüsiline või psüühiline defekt, siis tuleb need defektid jagada ravitavateks ja ravimatuteks. Ravimatute defektide puhul ei saa eesmärgiks seada isiku tervenemist ning seetõttu tuleb välja pakkuda vahendeid, et teha isik kahjutuks . Kahjutuks tegemise vahendeid on välja pakutud bioloogilise lähenemisviisi poolt väga mitmesuguseid. Üks on surmanuhtlus, teine eluaegne vanglakaristus või kui on tegemist haigetega, siis kas kinnipidamine või vaimuhaigla. On pakutud välja veel mitmesuguseid teisi vahendeid - kirurgilist vahelesegamist (on kasutatud ajukirurgiat - lobotoomiat, kus lauba piirkonnast katkestatakse mõningad juhtteed ajupiirkondade vahel ja on saanud ka väga olulist efekti). Kirurgilise vahelesegamisega on kurjategijate puhul saavutatud aktiivsuse langus kuritegeliku käitumise osas, kuid tagajärjeks on ka muu käitumise aktiivsuse langus. Seetõttu ei ole see tõenäoliselt mitte kõige optimaalsem lahendus. USA-s kasutati seda küll 60-ndate eksperimentaalselt. On ka mitmeid vahepealseid teid, näiteks seksuaalkuritegude puhul. USA-s pakuti välja valikuvõimalus - kas lubada kastreerimist või minna kinnipidamiskohta ja siis vabatahtlikult süüdimõistetud valisid neile sobiva kahjutukstegemise viisi. Püstitub küsimus inimõigustest.
    California was the first U.S. state to provide a legislative mandate for chemical castration. The 1996 California law mandated the use of chemical castration as a precondition for parole in the case of repeat sexual offenders of all types . In 1997, three other states passed similar laws so that a total of four states currently permit chemical castration. Research has shown that Depo Provera does reliably reduce sexual desire in men although it has no impact on sexual desire in women and therefore cannot be used in the case of female sex offenders. Chemical castration greatly reduces recidivism rates of paraphiliacs from over 90% to approximately 2%. This means that a chemically castrated parolee need not be considered as any more of a threat to victim populations, including children , than the male population in general.
    Kui püüda anda bioloogilisele lähenemisviisile mingit üldist hinnangut , siis see lähenemisviis on olnud küllaltki kõikuva hinnangu osaline. See lähenemisviis sai laiema leviku möödunud 19. sajandi lõpus. Algul lükati bioloogilise lähenemisviisi esindajate vaated küllaltki kergelt ümber. Ning pikaks ajaks kadusid bioloogilised vaated tunnustatud kriminoloogilise uurimise teemade hulgast, kuid kui vaadata käesolevat sajandit, siis 80-ndate lõpus, 90-ndatel aastatel on bioloogiline koolkond küllaltki oluliselt oma positsioone taastanud. On kaks riiki, kus bioloogilisse koolkonda on suhtutud väga negatiivselt: NSVL -is, kus algul sai bioloogiline koolkond küllaltki tugeva aluse, tehti poliitiline otsus, et kuritegevus on kindlasti sotsiaalne nähtus ja kuritegevus on “paha kapitalismi” selge tunnus ning otsustati, et kuritegevuse puhul ei ole lubatud rääkida bioloogilistest põhjustest ning puhtadministratiivselt keelati kõik bioloogilised uuringud. USA-s jõuti bioloogilise koolkonna või bioloogiliste vaadete eitamiseni teist teed pidi - bioloogiline lähenemisviis kippus jõudma tahes-tahtmata järeldusele, et bioloogiliste erisuste üheks võimaluseks on rassilised erisused ja ka need seletavad erinevusi kuritegevuse tasemetes erinevates elanikkonna kihtides ning kuna USA-s on rassilised probleemid küllaltki delikaatne teema, siis tõmmati kriips peale igasugustele avalikele toetustele taolistele programmidele st. ei pakutud rahalisi võimalusi. Selline rahalistele võimalustele kriipsu "pealetõmbamine" tähendas täpselt sama, mis administratiivne keelamine endises NSVL-s. Bioloogilised uurimused kuritegevuse toimumise põhjustest kadusid pikaks ajaks rohkem tunnustatud ja levikut omavatest kriminoloogilistest töödest.
    Euroopas jäid bioloogilised ideed siiski püsima. Negatiivset suhtumist võis leida Saksamaal, sest Saksamaal oli oma raske probleem - fašism. Bioloogilise suuna rakendamine ja kasutamine kippus minema sellisesse suunda, kus ta sai pideva avaliku negatiivse hinnangu osaliseks .
    Euroopas jäid bioloogilise suunaga tegelema sellised maad nagu Prantsusmaa, Belgia, oluliselt Taani, aga kui vaadata praegust momenti , siis bioloogilise koolkonna vaadete tunnustamine on jõudnud tagasi Venemaale, kus on täiesti arvestatavaid bioloogilise koolkonna suunas tehtavaid uuringuid . Samuti publitseeritakse ja avaldatakse bioloogilise suuna töid USA-s juba täiesti arvestataval tasemel.
    Bioloogilised teooriad kriminoloogias
    2.1 Frenoloogia – F. J. Gall , C. Lombroso
    Üheks esimeseks bioloogilise lähenemisviisi süstematiseerimise katseks võib pidada sellist õpetust nagu frenoloogia. Frenoloogia on oma juurtega pärit 18 -19 sajandi vahetusest. Teooria põhineb arusaamal, et kuna käitumise juhtimise protsessid toimuvad ajus, siis käitumise iseärasused peaksid peegelduma aju iseärasustes ning see peaks kuidagi puht väliselt olema ajus kajastatud. Arvati, et kui aju on erinev, siis mõjutab ta kuidagi teda ümbritsevat pealuud . Hakati otsima seoseid inimese kolba kuju ja tema käitumise vahel.
    Frenoloogia töötas välja terve seisukohtade kompleksi, kus erinevatele koljupiirkondadele anti erinevaid kuriteoliike mõjutav tähendus. Ühte sellist frenoloogia õppevahendit võib näha ka TÜ Ajaloomuuseumis, kus on see kolp olemas koos pealkirjadega, mis piirkond missugust hälvet võiks tähendada.
    Aja jooksul on selgunud, et need seosed on ilmselt siiski märksa keerukamad. Praegugi nõustume seisukohaga, et meie käitumist reguleerib oluliselt peaaju, aga seda, mis seal peaajus toimub, seda on väga raske väliskuju järgi hinnata. Praegu hakatakse uuesti seda uurima .
    The destructiveness center, for example, which is located right behind the ear above Darwin's point, is pronounced in 17% of criminals . Other bumps, in the back of the head, turned out to be pronouncements of the Amygdala and Hippocampus, where tumors are associated with criminal behavior (as in the Texas sniper, Charles Whitman). The general rule is that any abnormality in the back of the head is bad, "back is bad".
    2.2. Cesare Lombroso.
    Cesare Lombroso (1836 - 1909) oli Itaalia vanglaarst ja kohtumeditsiini professor . 19. sajandi keskpaik ja lõpp oli empiirilise loodusteadusliku uurimise algusaeg ning Lombrosol tekkis huvi kurjategijaid empiiriliselt uurida, et välja selgitada, kas kurjategijad omavad midagi spetsiifiliselt erinevat, mis eristaks neid normaalsetest inimestest. Samast perioodist pärineb ka Darwini õpetus looduslikust valikust ja Lombroso mõttekäik liikus suunas, et kui vaadata loomariiki, siis võib seal näha sellist käitumist, mida me inimese puhul peame kuritegelikuks. Sellega vastanduti Beccaria ja Benthami klassikalistele teooriatele. Ta hakkas püüdlema selle poole, et leida, kas kurjategijatel on näha sarnasusi loomadega . Kui inimene "õigete” inimeste hulka ei kuulu (st. ei pea kinni normidest), siis äkki peaks seda olema ka väliselt näha. Ta leidis mitmesuguseid atavisme, kuid siiski ei olnud neid mitte kõigil kurjategijatel ning ta hakkas otsima muid väliseid füüsilisi tunnuseid, et seletada ära, mille poolest need teised on spetsiifilised ning panevad toime kuritegusid.
    Kui jälgida Lombroso vaadete arengut, siis pakkus ta alguses välja idee, et kõik kurjategijad peaksid olema kuskil loomariigi ja normaalse inimese vahepeal , kuid uurimise tagajärjel leidis ta, et kõigil kurjategijatel ei ole võimalik leia neid bioloogilisi tunnuseid.
    Tunnused, mida uuriti, olid nt kolbad. Ferri viitab Lombrosole: normaalsete kolpadega mehi oli kolm korda rohkem sõdurite hulgas kui kurjategijate hulgas. Nende meeste hulgas, kellel esines korraga mitu anomaaliat, oli kolm korda rohkem kurjategijaid kui sõdurite hulgas. Polnud ühtegi sõdurit, kellel oleks üle 5 anomaalia. See selgitab, et kurjategijate hulgas on suurem hulk kolpade anomaaliaid, kus juures 50-60% kurjategijatest on mõned anomaaliad , 1/3-l palju anomaaliaid ja 1/10 on selles aspektis normaalsed.
    Sellest tulenevalt hakkas ta siis hilisemal perioodil jagama kurjategijaid erinevatesse rühmadesse, kus siis ainult üks osa olid kaasasündinud kurjategijad.
    Kurjategijate tüübid:
  • sündinud kurjategija (born) – moraalne imbetsill ja epileptik. Seos kuritegevuse ja epilepsia vahel on seotud rohkem päritolu kui identiteediga. Epilepsia esindab perekonda, mille liigid on moraalne hullus ja kriminaalsus.
  • kirekurjategija (by passion ) – seotud hoogsuse ja raevukusega, altruismist kihutatud. Siin on enamasti naised, sellest 91% on tapmised.
  • hullunud kurjategija (insane) – kleptomaanid, nümfomaanid, joodikud , pederastia. Kuritegu on põhjustatud ajudefektist, mille tõttu ei osata head halvast eristada. Neid inimesi peeti tõeliselt hulludeks ja vastutusvõimetuteks.
  • aeg-ajalt kurjategija (occasional) – kõige laiem kategooria, seal on omakorda 4 tüüpi:
    • pseudokurjategija – enesekaitse (seadust ei riku, juhuslik olukorda sattumine )
    • kriminoloid – epileptoidid, kellel on nõrgem haigus ja kurjategijad on lihtsalt nõrgad isiksused, kes sooritavad kuritegusid olukorra tõttu
    • harjumuslikud kurjategijad – süstemaatiline rikkumine, tegevusala neile (vargus, pettus, süütamine, võltsimine, väljapressimine jne)
    • epileptoidsed kurjategijad – kannatavad epilepsia all.

    Lombroso vaatles väga palju erinevaid kuritegusid ja nende tekke põhjusi. Algselt tuli ta kriminoloogiasse meditsiini poole pealt ja pakkus välja bioloogilised seletused , aga mida enam ta kuritegevusega tegeles, seda rohkem hakkas ta omaks võtma, et kuritegevusel on kindlasti ka palju mitmeid muid, sh ka mitmesuguseid sotsiaalseid põhjuseid.
    Hoolimata tema paludest ekslikest seisukohtadest ei ole võimalik mitte näha Lombroso positiivset rolli kriminoloogia ajaloos. Ta oli üheks esimeseks praktikuks, kes süstemaatiliselt püüdis kurjategijaid empiiriliselt uurida. Lombroso jõudis selleni, et puhta abstraktse aruteluga ei ole võimalik jõuda mingite reaalsete tulemusteni, selleks et reaalset tulemust saada, on vaja teha empiirilisi uurimusi.
    Lombroso atavismid :
    • Nina on varastel lame, kõver; tapjatel kongus või nokataoline.
    • Pea suurus ja kuju
    • Näo assümeetria
    • Silmade defektid ja iseärasused
    • Lõualuu ja põsesarnade liigne suurus
    • Ebaharilikud sõrnad
    • Jne

    Lombroso käsitles ka muid faktoreid:

    Ta kasutas algset valedetektorit.
    2.3. Enrico Ferri.
    Enrico Ferri ( 1856 -1929) – itaalia kriminoloog, Lombroso õpilane ja mõttekaaslane
    Ferri faktorite teooria
    Füüsikalised faktorid on kliima temperatuur jne. Alguses võib tekkida imestus , et miks need peaks siis kuritegevust mõjutama. Kuidas füüsilised faktorid mõjutavad kuritegevust? Kindlasti on vähemalt selles osas kuritegude toimepanemine seotud füüsilise faktoriga, et kas on valge või pime. Valges pannakse toime ühtesid kuritegusid, pimedas teisi. Samuti on temperatuuriga, samuti mõjutab kuritegude toimepanemist asjaolu, kas on väljas vihmane või paistab päike. Neid asjaolusid on uuritud ja on leitud objektiivseid seaduspärasusi. Ferri jaotas algul väga kindlalt kõik faktorid kolme rühma, aga siis mõistes, et nende füüsiliste faktorite suhtes preventsiooni vahendeid ei leidu, ta ei hakanud hiljem neid füüsilisi faktoreid enam eraldi välja tooma .
    • Antropoloogilised faktorid (indiviidi iseloomustavad faktorid)
    Antropoloogilised faktorid kujutavad endast kuritegevuse bioloogilist determineeringut. Selle suuna juures tõi Ferri välja järgmise uue mõistete paari: isiku ühiskonnaohtlik seisund ja sotsiaalse kaitse vahendid. Kui tavaliselt klassikaline kriminaalõiguse koolkond vaatleb paaris mõisteid - kuritegu ja karistus, siis Ferri pakkus, et nende mõistete asemel oleks hoopis ratsionaalsem rääkida isiku ühiskonnaohtlikust seisundist ja sotsiaalse kaitse vahenditest. Ferri leidis, et meile ei peaks huvi pakkuma mitte see, et kas oli nüüd tegemist kuriteoga, vaid see, et kas antud isik on ühiskonnale ohtlik st., kas ta võib käituda nii, et tekitab ühiskonnale mingeid kahjulikke tagajärgi või mitte. Samas ei saa sellistel juhtudel rääkida enam karistusest kuna karistamine ei ole loogiline vahend ohule reageerimiseks.
    Probleeme tekib sellega, et kuidas otsustada selle üle, kas on tegemist isiku ühiskonnaohtliku seisundiga või mitte, st. kuna inimene on ohtlik ja mille järgi me seda otsustame. Üks võimalus on otsustada ainult tema kuritegeliku käitumise järgi, kas isik on ühiskonnaohtlik või mitte ning sõltuvalt sellest, kas ta on kuriteo toime pannud ja millise kuriteo ta toime pani, hinnata tema potentsiaali tulevikus kuritegu toime panna.
    Teine tee oleks arvestada isiku ühiskonnaohtlikku seisundit juba enne kuriteo toimepanekut. Sellise seisundi tuvastamine on kindlasti keeruline. Siin tuleks mõelda, kas oleme nõus mingisugusel etapil, kui isiku ühiskonnaohtliku seisundi tuvastamise tõenäosus on juba kõrgem kui praegu mõnede uuringute puhul saavutatud umbes 70%, antud isiku suhtes midagi ette võtma.
    Kas me oleme valmis üleüldse rakendama mingitel tingimustel mingeid vahendeid tõenäosuslike andmete alusel? Kui me vaatame oma kohtusüsteemi, kas meil seal on 100%-line garantii? Ei ole, pole võimalik leida sellist kohtusüsteemi, kus oleks 100%-line garantii - kõik kohtusüsteemid teevad vigu. Tõepoolest standardviga on see, et süüdiolev isik mõistetakse õigeks - see on sagedane viga, aga esineb kahjuks mitte nii väga harva ka seda, et süüdi mitteolev isik mõistetakse süüdi. Kohtute puhul me talume seda st. me lubame kohtutel isikuid karistada , kuigi me teame seda, et kohtud karistavad ka süütuid inimesi - isikuid, kes ei ole süüdi.
    See, milliseid sotsiaalse kaitse vahendeid, peetakse aktsepteeritavateks sõltub oluliselt isiku ühiskonnaohtliku seisundi tuvastamise viisist. Juhul kui see tuvastatakse kuriteo toimepanemise järel, siis peetakse aktsepteeritavaks rangemaid vahendeid k.a. tähtajatut (kuni paranemiseni) vabadusekaotust. Juhul kui ühiskonnaohtlik seisund tuvastatakse enne kuriteo toimepanemist, siis pakutavad vahendid on oluliselt leebemad .
    Kaasaegses kriminaalõiguses on oluliselt tunda sotsiaalse kaitse vahendite kontseptsiooni pooldajate mõju. Näiteks: tingimisi enne tähtaega vabastamine jne.
    • Sotsiaalsed faktorid
    Ferri pööras suurt tähelepanu ka sotsiaalsetele teguritele. Tähistamaks vahendeid, mida on võimalik kasutada vältimaks sotsiaalsete faktorite negatiivset mõju, pakkus Ferri välja uue mõiste - karistuse ekvivalendid.
    The elimination of the causes of crime , 113 –
    Economic remedies for crime, 114 –
    Drink and crime, 116 –
    Drunkenness an effect of bad social conditions, 120 –
    Taxation of drink, 120 –
    Laws against drink, 121 –
    Social amelioration a substitute for penal law, 121 –
    Social legislation and crime, 122 –
    Political amelioration as a preventive of crime, 124 –
    Decentralisation a preventive, 126 –
    Legal and administrative preventives, 128 –
    Prisoners ' Aid Societies, 130 –
    Education and crime, 130 –
    Popular entertainments and crime, 131 –
    Physical education as a remedy for crime
    Karistuse ekvivalente (suunatud ühiskonnale, mitte indiviidile ja nende eesmärk on saavutada sama mis läbi karistuse) grupeeris Ferri ühiskondliku tegevuse sfääride kaupa:
    1. ühiskonna majandusliku korralduse sfäär - kaubanduse vabaduse tagamine, emigreerumise vabaduse tagamine, tulumaks (et toetada riigi sotsiaalprogramme), tollimaksude vähendamne, alkoholiaktsiis jne;
    2. ühiskonna poliitilise korralduse sfäär - mõttevabadus, rahva huvide arvestamine, esindusorganite funktsioneerimine , vabad valimised;
    3. teadus sfääris - kurjategijate tabamise ja kuritegude avastamise vahendite arenemine, vara kaitse arenemine, raamatupidamise areng;
    4. tsiviil- ja haldussuhete sfäär;
    5. religiooni sfäär - kiriku õiguste vähendamine, vaimulikele abiellumise õiguse andmine;
    6. perekonnasuhete sfäär - lahutuste lubamine, prostitutsiooni reguleerimine;
    7. kasvatuse sfäär - õpetajate olukorra parandamine, mängupõrgute keelamine.
    Seda mudelit on nii palju kuritarvitatud, seega on sel väga halb maine juures (selle võtsid omaks natsid ja NL – isiku ühiskonnaohtlikkuse tuvastamine ENNE esimest kuritegu). Siiski kasutatakse neid praegu – nt psüühiline sundravi, tingimisi karistamine, vahi alla võtmise puhul (põgenemise tõenäosus jne), kainestusmajja paigutamist, ennetähtaegsel vanglast vabastamisel.
    2.4. Kromosoomanomaaliate uuringud (XYY)
    Henry Goddard (USA psühholoog; IQ test inglise keelde; tugev päriliku mõju tunnistaja) - sugupuu uuringud (Kallikaki sugupuud ). Nendele ei tasu väga palju rõhku panna. Nad on sageli lihtsalt huvitavad kõrvalt vaadata.
    Kuritegeliku käitumise aspektist on kõige rohkem uuritud sugukromosoomianomaaliaid. Meestel on sellise anomaalia korral üks y- kromosoom lisaks (tavaliselt naine xx, mees xy, taolise anomaaliaga mehel xyy). Normikuulekate hulgas on taolise anomaalia esinemissagedus ca 0,2%, kurjategijate hulgas ca 2%.
    Spetsiifilist genotüüpi puudutav. Kas kurjategijate genotüüp erineb mingit moodi mitte kurjategijate omast? Me tahame saavutada seda, et inimesed ei kardaks geeniuuringus osaleda. KA on sellised, kus uuritakse kui palju kromosoome üldse on jne. Tavaliselt on meil 23 kromosoomi paari.
    2.5. Kaksikute uuringud.
    Nende uuringute puhul võrreldakse ühemunarakukaksikuid - monosügootsed kaksikud (MZ) ja erimunarakkude kaksikuid - disügootsed (DZ). Uuringu sisuks on see, et uuritakse kaksikutepaare ja püütakse leida, kas paarid käituvad ühtemoodi. MZ kaksikute puhul langeb käitumine kokku 60%, DZ kaksikute puhul 30 %. Kokkulangeva käitumise protsent on erinevatel uuringutel olnud erinev. Eeldatakse, et kaksikute paari liikmed elavad omavahel võrreldes võrdsetes sotsiaalsetes tingimustes. Ja seega sotsiaalsete faktorite mõjust põhjustatud käitumise kokkulangevus peaks nii MZ kui DZ kaksikutel olema samasugune . Bioloogiline sarnasus on aga MZ ja DZ kaksikutepaaridel erinev - MZ kaksikud on geneetiliselt praktiliselt identsed, DZ kaksikud erinevad geneetiliselt teineteisest sama palju kui tavalised vennad. Bioloogilise koolkonna vaadete toetajad teevad siit järelduse, et kui sotsiaalsed tingimused ei saa käitumise kokkulangemise sageduse erinevust MZ ja DZ kaksikute paaride vahel seletada, siis tuleb uskuda , et DZ kaksikute vähem kokkulangev käitumine peab olema seletatav sellega, et nad on bioloogiliselt erinevamad. Bioloogiliste vaadete kriitika ründab taolist järeldust väidetega, et sotsiaalne keskkond ei ole kaksikutel võrdne, eriti DZ kaksikute puhul.
    ÜM: KM:
    Sotsiaalselt: 100% 100% (üleskasvanud ühes keskkonnas)
    Geneetiliselt: 100% 50%
    Kuritegevuslik käitumine: 60% 30%
    Kaksikute uuringut on kõvasti rünnatud, nt sellepärast, et kaksikute arv, keda uuriti oli algselt väike. Teine rünnak on see, et väidetakse, et küsitav on kas seda võib ikka uskuda, et KM kaksikutel sotsiaalne keskkond on samavõrra sarnane ÜM kaksikutel. See sotsiaalne keskkond, mis mõjutab, on sama, aga kui see mõjutus on erinev, siis see peab tulema siiski sealt bioloogilise poole pealt. Bioloogiline baas määrab paljugi ära, millised sotsiaalsed tingimused isikut mõjutama hakkavad. Ei ole võimalik uurida, et mis siis saab, kui sotsiaalse keskkonna poole pealt ei oleks võrdsed tingimused, kui nad ei kasvaks üles ühes keskkonnas. Üldjuhul nad kasvavad ühes keskkonnas. Aga kui nad peaksid üles kasvama erinevas keskkonnas, siis need tulemused oleksid küllaltki kõnekad. Mis ikkagi juhtub, kui sotsiaalsed tingimused oleksid erinevad. Kuidas bioloogilised faktorid mõjutavad siis, kui sotsiaalsed tingimused pole sarnased? Lapsendamine . Pole eriti sagedane, et kaksikuid lapsendataks erinevatesse peredesse, seega uurimusmaterjal on väga väike.
    2.6. Lapsendatute uuringud
    Selles uuringute rühmas püütakse välja selgitada, kuidas mõjutab lapsendatud lapse tulevast käitumist asjaolu, et tema bioloogiline vanem on toime pannud kuriteo, või et tema lapsendaja on toime pannud kuriteo. Lapsed jagatakse taolistes uuringutes nelja erinevasse rühma:
    a) ei bioloogilised vanemad ega lapsendajad ei ole toime pannud kuritegu (sellisel juhul 10,5% lapsendatutest panid toime kuriteo);
    b) bioloogiline vanem on toime pannud kuriteo kuid lapsendajad ei ole toime pannud kuritegu (sellisel juhul 21,5% lapsendatutest panid toime kuriteo);
    c) lapsendaja on toime pannud kuriteo kuid bioloogilised vanemad ei ole toime pannud kuritegu (sellisel juhul 11,5% lapsendatutest panid toime kuriteo);
    d) nii lapsendaja kui bioloogiline vanem on toime pannud kuriteo (sellisel juhul 36,2% lapsendatutest panid toime kuriteo).
    Erinevad uuringud on andnud küll erinevaid tulemusi kuid kõigil juhtudel on bioloogilise vanema kuritegelik käitumine olnud oluliseks lapsendatu kuritegeliku käitumise tõenäosust mõjutavaks teguriks .
    Negatiivsed kuritegevust soodustavad faktorid võimendavad teineteise toimet.
    Probleemid: kas on põhjust arvata, et bioloogilise vanema mõju on ainult geneetiline? Jah kindlasti neilt tuleb ka geneetiline mõju, kuid me ei saa olla kindlad, et see mõju n puhtalt geneetilised. On ka muud mõjud.
    2.7. Autonoomse närvisüsteemi funktsioneerimise erisused.
    Palju uuringuid on tehtud naha elektrijuhtivusega. Keskmistel kurjategijatel on naha elektrijuhtivus väiksem kui normikuulekatel. Kurjategijatel esineb harvemini spontaanseid naha elektrijuhtivuse tõuse. Väliste ärritajate mõjul hakkab kurjategijatel naha elektrijuhtivus tõusma kuid tõuseb aeglasemalt kui normikuulekatel. Erutuse raugemine toimub kurjategijatel pikema ajavahemiku vältel, normikuulekatel kiirelt.
    Bioloogia poole pealt veel:
    IQ ei ole mitte 100% kaasasündinud. IQ puhul ei saa neid mõjusid muidugi eitada. On võrreldud lastel ja lastevanematel IQ tasemeid. Bioloogiliste vanematega on sarnasused tugevamad kui lapsendajatega. Keskmised tulemused on need, et kurjategijate keskmine IQ on madalam, kui normikuulekatel.
    Tüübil oli ajukasvaja , mis ära opereeriti ja siis oli tüüp jälle normikuulekas. Mõne aasta möödudes aga hakkas jälle vigureid tegema. Opereeriti jälle kasvaja ära, oli taas normaalne.
    2.8 Biokeemilised uuringud
    Serotoniini sisaldus ajuvedelikus on kurjategijatel keskmiselt madalam kui normikuulekatel. Serotoniini sisaldusega seotakse inimese võimet tunda ennest hästi ja õnnelikult. Ollakse veendunud on serotoniini sisaldus inimese organismis on ette määratud bioloogiliselt. Samuti on leitud, et naiste kuritegelik aktiivsus on kõrgem vahetult menstruatsioonile eelnevatel päevadel.
    Üks serotoniinitase on veres ja teine on ajus. See, mis veres on, see ajju minna ei saa. Peab ise ajus tekkima . Tavaliselt ajus madalam ja veres kõrgem.
    Uurijad on leidnud ka seda, et parim ennustav faktor (kui lapsed on 7a) agressiivsuse, noorsookuritegevuse, hilisema kuritegeliku vägivalla suhtes on pliimürgitus. Teine on aneemia (seos kaltsiumi, tsingi ja teiste vajalike mineraalide puudusega), mis on seotud toksiinidele vastuvõtlikkusega.
    Sotsioloogilised mikrotasandi teooriad kriminoloogias
    3.1. Edwin Sutherland , diferentseeritud assotsiatsioonide teooria; valgekraede kuritegevus
    Peamine esindaja on Edwin Sutherland (1883 – 1950). Teooria on kirjas kõikides õpikutes. Teooria väidab, et nii kuritegelik kui delinkventne (hälbiv) käitumine on seotud kuritegude toimepanemise tehniliste võtete ja seaduserikkumist soodustavate motiivide, püüdluste, ratsionaalsete seletuste ja hoiakute omandamisega (vastastikuse mõjutamise protsess). Kahjuks on teoorias , nagu ka uuringutes, vähe tähelepanu pööratud sellele, mida omandatakse.
    Teooria põhiolemus seisneb selles, et väidetakse, et kuritegelik käitumine on õpitud käitumine.
    On võimalik kokku võtta 9 põhiteesina ja need on:
    1. kuritegelik käitumine õpitakse (mitte ei pärita)
    2. kuritegelik käitumine omandatakse teiste inimestega suhtlemise teel (verbaalne, eeldab žeste)
    3. kuritegeliku käitumise õppimine toimub põhiliselt väikestes gruppides ( impersonaalne suhtlemine mängib väikest rolli; samas teleka puhul suheldakse ju inimestega ekraani taga)
    4. kuritegeliku käitumise õppimine hõlmab nii tehniliste võtete õppimist kui ka motivatsiooni ja hoiaku õppimist;
    5. kriminaalseadus fikseerib selle kas õpitud käitumine on kuritegelik või mitt (USA ja kultuurikonflikt )
    6. inimesest saab kurjategija, kui seaduse rikkumist soodustavad väärtused on temas ülekaalus ( kummad assotsiatsioonid on ülekaalus; neutraalsetel pole üldse mõju)
    7. rikkumist soodustavate väärtuste olemine ülekaalus sõltub:
    · kontaktide prioriteedist; - varasematel kontaktidel on tugevam mõju ( nooremana kergemini mõjutatav) - vanemad
    · kontaktide sagedusest;
    · kontaktide kestvusest;
    · kontaktide intensiivsusest – austatakse – tanel padar
    Eelnevatel kontaktidel on tugevam toime kui järgnevatel. Mida sagedasem ja kestvam kontakt, seda tugevama mõjuga see on. Sama kestvus ja sagedus omavad erinevat toimet. Oluline on see millist prestiiži või tähendust omab isik, kellega kontakt on, teise isiku suhtes. Peale intensiivsuse on teisi tegureid võimalik hinnata väliselt ja objektiivelt. Intensiivsust saab vaadata selle järgi, kas isik läks kuritegu sooritama kontakti mõjust ajendatuna või mitte;
    8. kuritegeliku käitumise õppimise protsess toimub nagu iga teine õppimise protsess ning sel on olemas samad tunnused ja mehhanismid (see tähendab, et ei piirduta imiteerimisega, vaid ka nt assotsieerumine läheb sinna)
    9. ehkki kuritegelik käitumine on üldiste väärtuste ja vajaduste väljenduseks ei saa neid selle abil seletada, sest sellega on põhjendatud ka mittekuritegelik käitumine. Vargad üldselt varastavad raha pärast, aga ausad inimesed töötavad ka raha pärast.
    Viimase teesiga eristab see teooria end teistest teooriatest, mis väidavad, et mingid vajadused panevad kuritegu toime panema . Üldised normid ja vajadused suunavad ju ka normikohasele käitumisele.
    Antud teooriat on rünnatud sellega, et seda pole võimalik empiiriliselt tõestada. Peamiselt on probleemiks kontaktide intensiivsuse määramise võimatus.
    3.2. Thorsten Sellin, kultuurikonflikti teooria
    Esindajatest on oluline nimi Thorsten Sellin. Teooria lähtub sellest, et maailmas on palju erinevaid kultuurisüsteeme. Seoses mobiilsuse kasvuga eri kultuuride kokkupuuted sagenevad ja tekib rohkem ka kultuurikonflikte.
    Sellin vaatles kolme skeemi:
    1. kahe kultuuri piirimail toimuv kultuurinormide konflikt;
    2. juhud , kus ühe kultuuri normide territooriumile kehtestatakse teise kultuuri normid;
    3. juhud, kus ühe kultuuri normide kehtivuse territooriumile asub elama teise kultuuri normide kandja.
    Sellin vaatles kuidas need kultuurikonfliktid võivad leida väljenduse hälbivas käitumises. Variandid:
    1. ühe kultuuri normide poolt lubatud käitumine on teise kultuuri poolt kriminaliseeritud, kusjuures isik ei ole teadlik sellisest kultuurinormide konfliktist. Selline konflikt on kõige järsem konflikt ning loodetavasti juhtub selliseid konflikte järjest vähem (see on nn. esimese põlvkonna konflikt);
    2. isik on juba teadlik teise kultuuri normidest ja konfliktist enda harjumuspäraste kultuurinormidega, kuid isik on stressis kuna ta ei saa käituda nii, et ta täidaks mõlema kultuuri norme ning seeläbi võib suureneda isiku vägivaldsus jne (see on samuti nn. esimese põlvkonna konflikt); (truudusetusele vägivallaga vastamine)
    3. järgnevate põlvkondade probleemiks on see, et oma sotsialiseerumise perioodil elavad nad nii, et kodus on ühed ja väljaspool kodu teised normid. Isik ei võta kumbagi normistikku omaks ning seetõttu jääb isiku käitumine sellises situatsioonis kultuurinormide poolt täielikult määratlemata ja ta rikub tihti mõlema kultuuri norme (see on nn. järgneva põlvkonna konflikt).
    Näiteid leiab kultuuriantropoloogide töödest (nt Labradori indiaanlased ja valged inimesed – õigusnormidele allutatakse inimgrupid, kellel pole neist enne ettekujutustki olnud  konfliktid; Alžeerias Prantsuse kriminaalkoodeks ja truudusetu naise tapmine kui tava). Kultuurikonfliktid on sotsiaalse diferentseerumise protsessi tagajärg, kuna diferentseerumise tagajärjeltekib lõputu arv sotsiaalseid gruppe, kes kõik määratlevad elusituatsioone ja interpreteerivad sotsiaalseid suhteid omamoodi ega tunne üldse või tunnevad valesti teiste gruppide sotsiaalseid väärtusi. Seetõttu kaasneb homogeense, hästi integreeritud kultuuri muutumisega heterogeenseks ja desintegreerituks konkliktisituatsioonide arvu kasv. Ning vastupidi, integratsiooniprotsess vähendab konfliktisituatsioonide arvu. Selliseid kultuuri muutumise tingimustes tekkivaid konflikte tuleb eristada konfliktidest, mis tekivad erinevate kultuurisüsteemide kokkupuutel. Igal juhul hindavad teise grupi liikmed erinevate kultuurikoodeksite konflikti haaratud isiku käitumist kui teatud määral ebanormaalset.
    Mõned uurijad kontsentreerivad kultuurikonflikte uurides tähelepanu mõjule, mida niisugune konflikt osutab konkreetse isiku käitumisele. Selline lähenemine on omane psühhiaatriale, biograafilist meetodit kasutavale sotsioloogiale. Need teadlased vaatlevad taolist konflikti kui isikule sisemiselt omast. L. Wirth väidab kategooriliselt, et kultuurikonflite võib vaadelda delikventsust mõjutava faktorina ainult sel juhul, kui isik tajub seda konflikti ja tegutseb nii, nagu seda konflikti ei eksisteeriks. Kultuurikonflik on psühholoogiline konflikt, kuid erinevad distsipliinid mõistavad selle iseloomu erinevalt. Psühhiaatrid- Freudistid vaatlevad seda kui võitlust teadvuse süvakihtides peituvate ja väljendust nõudvate bioloogiliste tungide ja kultuuri poolt ettekirjutatud normide vahel. Kultuuri normid loovad kontrollimehhanismi, mis surub sellsied tungide väljendused alla ja tõrjub nad alateadvusse, kust nad murravad välja:
  • võttes sotsiaalselt lubatava vormi
  • sotsiaalseid norme rikkuva käitumisena (kui kontrollimehhanismis tekib häire)
  • neuroosina (kui kontrollimehhanism töötab liiga hästi).
    Sotsioloogid vaatlevad aga seda konflikti vastupidi, nimelt eelkõige antud isiku poolt omandatud antagonistlike käitumisnormide konfliktina. E. W. Burgess väidab näiteks, et psühholoogilist konflikti on alati võimalik seletada erinevate kultuuride vahelise konflikti terminites.
    Kui nõustuda selle seisukohaga, peaks piirduma nende uurimisega, kelle teadvuses kajastuvad erinevate kultuuride normid (biograafiline meetod). Aga siiski on ka sellised juhud, mille puhul sotsioloogilises mõttes konflik puudub, edukalt kirjeldatavad kultuurikonflikti terminites (New Jersey sitsiillane – konflikt kestab seni, kuni pole lõpule jõudnud uude kultuuri sissekasvamise protsess; alles seejärel on võimalik normide rikkumist vaadelda psühholoogilise konflikti terminites).
    Juhul kui kultuurikonflikti saab mõnel juhul vaadelda personifitseeritult või psühholoogiliselt, mõnel juhul aga ainult täiesti objektiivse konfliktina erinevate gruppide koodeksite vahel, siis on ilmne, et uurimine ei tohi piirduda ainult psühholoogiliste konfliktide uurimisega.
    Biograafilisele meetodile osutav eriline tähtsus tuleneb oletusest, et ühe isiku kogemus avab samal ajal tema grupi elutegevuse iseloomu ja et inimese harjumused on ühiskonnas eksisteerivate tavade väljenduseks. Selline on kahtlemata ainuke teaduslikku väärtust omav lähenemine. Sellelt aluselt võime jõuda teaduslike üldistusteni, kui uurida isikuid, kes:
  • on omaks võtnud erinevate gruppide vastuolus olevad käitumisnormid (ainult siin võib rääkida psühholoogilisest konfliktist) või
  • käitusid vastavuses käitumisnormidega, mille nad võtsid grupilt, kelle koodeks on vastuolus selle grupi koodeksiga, kes käitumist hindab (konfliktil puhtväline iseloom)
    Olulist rolli mängib ka kultuurinormide kandjate hulk ühiskonnas. Kui ükski kultuur ei domineeri , siis tekib probleem kuna pole teada millised kultuurinormid kehtivad.
    Eelpool vaatlesime me suuri süsteeme, kuid kultuurinormide konfliktid esinevad ka väikeste rühmade tasandil. Näiteks saab täheldada kultuurinormide konflikte isiku liikumisel ühest ühiskonna kihist teise.
    Kultuurikonfliktiteooriaid on püütud kasutada kohtutes kuritegu kergendava asjaoluna (USAs on õnnestunud päris palju). Näide: taanlanna jättis lapse vankriga kohviku akna alla ja läks ise sisse. USAs on rangem hoolsuskohustus, kuid Taanis on see aktsepteeritav. Taanlanna pääses karistusest. Korealanna Chong Sun France jättis oma lapse riiulisahtlisse mitmeks tunniks, kui ta tööle läks ja laps hukkus lämbumise tagajärjel. Ta mõisteti 20 aastaks vangi kuid vabastati kolme aasta pärast, kui Korea naised selgitasid korea lapsehoolduskultuuri. Puudub kindel reegel, kui kaugele kohus on valmis minema kultuurikonflikti osas. Ginteri arvates tunnistatakse kultuurikonflikti kergemini naiste suhtes.
    1997. a mõisteti 2 korea misjonäri süüdi, sest nad ajasid kurje vaime välja nii, et see viis inimese surmani (peksid jalgade ja kätega kõhtu, mis pidi deemonid välja viima). Kohtunik mõistis nad süüdi mitte mõrvas, vaid tapmises ettevaatamatusest. 1983. a mõisteti jamaikalane süüdi naise tapmises (naine olevat nõid ja plaaninud teda nõidusega tappa). Pärast antropoloogi arvamust vähendas kohtunik karistuse tapmiseni ettevaatamatusest. 1968 indiaanlane – tema puhul analoogiline olukord ei õnnestunud, sest kohtunik ei lubanud eksperdil rääkida spirituaalsetest uskumustest indiaanlastel.
    Seoses vägisamisega ei läinud läbi kuubalase rõhumine 16a sõbralikkusele ja kanepisuitsetamisele, iraanlase rõhumine sellele, et keele tõttu ei saanud aru, et tüdruk oli vaimselt alaarenenud ja mustanahaline ei saanud rõhuda sellele, et neil ongi kombeks valjult rääkida ja ta ei saanud aru, et naine kartis ja polnud nõus.
    Kong Moua röövis pruudi endale. Kui naine ei protesteeri veenvalt , loetakse teda voorusetuks ja kui mees annab alla, loetakse teda nõrgaks. Pärast suguühet (millal naine peab ikka protestima) jõuavad vanemad abielus kokkuleppele. Selle traditsiooni valguses on naise protesteerimine rituaali osa. Kui aga naine ei tunne, et ta käituks rituaali osana , tunneb ta, et teda röövitakse ja vägistatakse. Kaitse tugines sellele, et mees ei saanud aru, et naine protesteerib tegelikult, mitte rituaali osana. Süüdi mõisteti ta ainult vabaduse võtmise osas.
    Kuidas muutub vastava piirkonna kriminaalne aktiivsus vastavalt sellele, milline elanikkond seal elab?
    Kuritegevus on suurem, kui mustade ja valgete osakaal on umbes 50:50. Kultuurikonfliktid ja ajatelg: ei oska täpselt öelda, kus ajateljel asutakse.
    3.3. Neutraliseerimise teooria
    Esindajateks on Sykes ja Matza. Nende seisukoht on, et ei ole põhjapanevat erinevust normikuulekate isikute ja kurjategijate poolt tunnustatavate normisüsteemide vahel. Mõlemad tunnustavad ühtesid ja samu ühiskonna norme.
    Oma seisukohta põhjendasid nad sellega, et kurjategijad näitavad kättesaamisel üles kahetsust st. kurjategijad ei ole rahul sellega, mida nad on teinud. Kriitika selle põhjenduse vastu tõstatab küsimuse kahetsuse siiruse pinnalt. Teine asjaolu, millega põhjendatakse seda seisukohta on see, et kurjategijad kipuvad ohvrite valikul lähtuma üldkehtivatest moraalinormidest. Ohvrid ja kurjategijad on tihti pärit ühest sotsiaalsest kihist.
    See teooria selgitab, miks alaealised panevad toime õigusrikkumisi sõltumata sellest, et nad ise peavad end suuremal või vähemal määral seotuks seaduse nõudmistega. Probleemi lahendus peitub faktis , et teatavat käitumist nõudvad sotsiaalsed normid on harva impertiivsel kujul väljendatud (süsteem on elastne). Elastsus on omane ka kriminaalseadusele (hädakaitse, süüdimatus, hädaseisund). Sykes ja Matze väidavad, et hälbiv käitumine seondub vastutust välistavate asjaolude ringi alateadliku laiendamisega, mida peab õigeks hälvik, kuid ei tunnusta õigussüsteem ega ka ühiskond tervikuna .
    Selliseid õigustusi määratletakse tavaliselt kui õiguserikkuja poolseid ratsionaalesid oma teo seletusi, kuid on alust arvata, et see on suunatud teoeelsele ajale, mitte hilisemaks süütunde ja süüdistuse tagasilükkamiseks. Hälvik muudab teo enda jaoks vastuvõetavaks või isegi nõutavaks. Nii ei kujuta ta aga opositsiooni, vadi pigem vabandavad hädavarest, kes arvab , et ta on rohkem teiste pattude ohver kui patune.
    Normi neutraliseerimise meetodid (Suhterland: seaduserikkumist soodustavad hinnangud ). Normi neutraliseerimine toimub:
    1. vastutuse eitamise teel s.o normi rikkumine ei toimunud minu vaba tahte vaid väliste asjaolude kohaselt ( piljardipall – halvad vanemad, sõbrad, subkultuur ); väiline asjuolu on siin kujunenud minevikus – nt väärkoheldud laps. Kaasaja juhtumitest võib siia tuua massiga normi rikkumisega – ega mina ei teinud, suur mass tegi kõike. Psühhodünaamilisest seisukohast võib isiku selline orientatsioon tähendada sügavat võõrandumist oma minast. Nähtav on seos hälbiva käitumise sotsioloogiliste seletuste ja humaanse õigusmõistmise süsteemi vahel. Õppides vaatlema end rohkem mõjutuste objektina kui aktiivselt tegutseva isikuna, valmsitab hälvik ette pinnast hälbivaks käitumiseks, vältides samal ajal vajadust üldkehtivatele normidele otseselt vastanduda.
    2. kahju eitamise teel s.o antud käitumine ei tekitanud kellelegi kahju ja sel juhul ei saa selline käitumine olla keelatud. Paljude riikide kriminaalsüsteem teeb vahet kuritegudel, mis on olemuslikult kuritegelikud ( mala in se) ja kuritegudel, mis on küll ebaseaduslikud, kuid ei ole amoraalsed (mala prohibita). Võib teha samasuguse vaheteo, kuid küsimus nihkub sinna, kas kellelegi tehti kahju. Seda võib tõlgendada mitmeti, nt vandalismi võidakse vaadata vallatusena (isikud võivad ka kergelt korvata oma kahju); autovargust võib vaadelda omastamisena või kampatevahelist kaklust eratlina või huvitatud poolte eelneval kokkuleppel toimuva duellina, mis pole ühiskonna kui terviku asi. Ähmastub seos teo ja selle tagajärje vahel. Tehakse laiendav suhtumine – kui nt on tegemist ainult ohustamisega, siis see ei vääri karistust; ka siin võib olla see, kui kannatanu on riik. KarS : suhtumine tegudesse, mis ei too kahju, on pehmem kuna neil on vähem kahjulik tagajärg samuti eriosa normidega, mille järgi paljude kuritegude puhul on nõutav kahjuliku tagajärje saabumine.
    3. kannatanu eitamise teel s.o puudub reaalne isik või kurjategija leiab, et selle isiku vastu suunatud käitumine ei tohiks olla keelatud (õiglane kättemaks või karistus, end pannakse tasuja rolli); - kui kannatanu muutub kurjategijaks (hädakaitse piiride ületamine); kui kannatanu on riik; spetsiifiliselt siin on need, kus kannatanu väärib kuritegu. Siia võib tulla riigi puhul see, kui kurjategija on mõnel hetkel väga vajanud riigi toetust, aga pole saanud, aga ka siis, kui tuntakse, et riik ei kasuta otstarbekalt. Nt Amigo juhtum – mis nad ülbitsevad turvamehega. Lähedane hädaseisund. See teooria töötab ka siis, kui rünnatakse oletatavaid või tegelikke homoseksualiste, vähemusgruppde esindajaid, samuti vandalismi puhul (ebaõiglasele õpetajale kätte maksmine ). Kui me tahame, et teadvustatud normid ja teiste isikute reaktsiooni ettenägemine reguleeriks käitumist, siis peavad nad olema nii või teisiti aktiviseeritud ja väga võimalik, et ebapiisaval ettekujutusel ohvrist on küllalt tähtis os aseliste protsesside tekkimise või mittetekkimise määramisel.
    4. hukkamõistjate hukkamõsitmine - kurjategija paneb kuriteo toime ja väidab, et politsei ja teised, kes tema käitumist kritiseerivad, pole ise sugugi ausad ja nad teevad ise ka pätti ning seega pole neil moraalset õigust süüdistada, sest hukkamõistjad ise pole paremad; nt tšetšeeni terrorist, purjus politseinik . Lähedane stigmatiseerimisele. Niisugune orientatsion kogu seaduskuuleka ühiskonna suhtes võib omandada erilise tähenduse, kui ta muutub domineeriva ühiskonna normide kehtestajate ja täitmise tagajate vastu suunatud küünilisuseks. Võib väita, et politsei on äraostetav, jõhker vms. õpetajatel on alalti lemmikud, vanemad valavad paha tuju lastele jne. mõningase liialdusega võib jõuda seisukohale, et tasu käitumisnormide järgimise eest (nt materiaalne heaolu) sõltub protektsioonist või vedamisest ja sellega muutub nende positsioon, kes alluvad seadusele, veelgi enam haavatavaks. Ei ole tähtis, kuivõrd põhjendatud on selline ühekülgne arvamus, vaid see, kuidas ta aitab tõrjuda ja kõrvale lükata normide rikkumisega seotud negatiivseid emotsioone. Tulemuseks on see, et hälvikul muutub kergemaks vähendada või mitte märgata oma käitumise ebaõigsust.
    5. tähtsamate asjaolude poole pöördumise teel s.o ma panin teo toime situatsioonis, kus olid kehtivad mitmed erinevad konfliktsed normid ja ma sain täita neist vaid ühte ning ühte täites pidin ma rikkuma teist. Siia sobib punase tule näide paremini kui vastutuse eitamise tee. KarSis kohustuste kollisioon , seos hädaseisundiga. Siin tuuakse ühiskonna nõudmised ohvriks väikese grupi nõudmistele. Vaatamata hälviku mitteallumisele domineerivale normatiivsele süsteemile, ei ole üldsegi mitte vajalik, et ta hülgaks selle süsteemi normid. Hälvik võib ennast vaadelda pigem isikuna, kes seisab dilemma ees, mis tuleb kahjuks lahendada seaduserikkumisega. Üht taolise situatsiooni aspektidest analüüsisid S. A. Stouffer ja J. Toby, kes uurisid isiklike ja ühiskondlike nõudmiste konflikti, konflikti sõpruse ja ühiskonna huvide vahel. Nende uuringute tulemused näitavad, et inimesi on võimalik klassifitseerida sõltuvalt sellest, millise valiku poole nad rohkem kalduvad. Kuid meie jaoks on kõige olulisem siin see, et mõnede normide kõrvalejätmine võib toimuda mitte sellepärast, et nendest on loobutud, vaid sellepärast, et teised normid, mida peetakse tungivamateks ja seonduvateks tähtsamate kohustustega, saavad ülekaalu. Tõepoolest, just see fakt, et hälvik peab tegelikuks nii ühtesid kui teisi norme, on dilemma ja sotsiaalsete rollide konflikti kontseptsiooni sisuks.
    Sõpruse ja seaduse nõudmiste vahelised konfliktid ja teised sellised dilemmad on juba ammu teadvustatud sotsioloogide poolt kui üks üldinimlikest probleemidest. Kui noor ka valib sõbrad, jääb see valik ikkagi arusaadavaks ka sellele, kes allub seadusele. Ebaharilikkus seisneb võib-olla vaid selles, et hälvik võib oma olulisi rikkumisi väga sageli vaadelda väikese grupi huvides toimepandavate tegudena. Kuid see on erisus ikkagi ainult astmes , mitte olemuses.
    Antud teooria puhul käib arutelu, kas neid neutraliseerimismeetodeid kasutatakse enne või pärast kuriteo toimepanemist. Kas kurjategija peab juba enne neutraliseerima üldkehtiva moraalinõude toime? Ühest küljest ollakse arvamusel, et see toimub ainult enne kuriteo toimepanemist, sest see on oluline selleks, et kurjategija saaks otsustada kuriteo toimepanemise kasuks. Kui otsustamine on siis tehakse seda alateadlikult, mitte arutlemise ja kaalumise tulemusena. Isik võtab juba kujunemisel omaks teatud põhjendused, millistes tegudes ei saa teda kurjategijaks pidada.
    On olemas statistika, et sarnased kihid on ohvriteks sagedamini. Seda saab kirjeldada nii neutraliseerimisteooriaga (teadlik valik – püütakse mitte minna üldise arvamuse vastu) ja rutiinsete tegevuste teooriaga (kõige tavapärasem tegevus mõjutab kuritegevust (kool-kodu-töö). Rutiinsete tegevuste teooria puhul esinevad koos kuritegevuse sihtmärk, motiveeritud kurjategija, ärahoidjate puudumine. RTT lisab kontrolliteooriale sobiva sihtmärgi. Sihtmärk on sobiv, olenevalt sellest, kui suur väärtus on ja kui suurt pingutust nõuab. Kuidas RTT jõutakse? 3 komponendi koosesimene sõltub meie rutiinsetest tegevustest. Kui palju vaeseid koos elab, on seal piirkonas väiksem kuritegevus!
    3.4. Stigmatiseerimise (märgistamise) teooria.
    Esindajaks on Lemert. Teooria väidab, et kuritegelik käitumine pole kuritegelik iseenesest, vaid saab selleks märgistamise kaudu riigi sunniaparaadi poolt. Samas on kuritegelik käitumine kui mõiste suhteline. Erinevad ühiskonnad kriminaliseerivad erinevaid käitumisi. Antud teooria absolutiseerib suhtelist aspekti kuritegevuse juures. Teooria kokkuvõetav teesidena.
    Kuidas märgid mõjutavad inimest? Psühholoogide uuring klassiõpetajate ja õpilastega, kus õpilased jagati suvaliselt tublid /nõrgad nimekirja. 6 kuu pärast olidki nn nõrgad nõrgemad ja nn tublid tublimad. Teine näide: tööandjatele saadeti CV-d ning igasugunegi vihje seal tõmbas tööotsija võimalused üsna nulli. Intervjuule kutsuti 24% neist, kellel oli suvaline vihje kuritegevusele, 12% neist, kes olid õigeks mõistetud ja 4% neist, kes oli süüdi mõistetud. Kolmas näide; isikud, kes väitsud, et kuulsid luulusid, saadeti psühhoneuroloogiahaiglasse, kus järgmisest päevast alates nad enam midagi ei kuulnud , kuid keskmiselt kulus 19 päeva vabanemiseks, samuti kirjutati nende märkmete tegemise kohta haiguslehele mingi märkus.
    Kuidas indiviidid end ise märgistavad? On tõdetud, et sisemise märgistamise puhul on sellel ühiskonnale suurem negatiivne mõju kui välimise märgistamise puhul.
    Teesid:
    1. Ükski tegu pole iseenesest kuritegu, vaid ta saab selleks kriminaalseaduse järgi;
    Karistusseadus paneb nad lihtsalt kirja, aga nad on ühiskonnas juba olemas. Aga tegelikult on piir kuskil mujal enamasti – raske leida seda.
    2. Kuritegelikuks käitumiseks tunnistatavate tegude piirid määratakse politsei poolt, selle aktiivse tegevuse läbi;
    Politsei aktiivset tegevust igale poole ei jõua (nt lapse vitsaga karistamine, võõras wifi ). Kas see on mõistlik?
    3. Inimene saab kurjategijaks mitte sellega, et ta rikub seadust, vaid stigmatiseerimise protsessis, mille kaudu võimud annavad talle sellise staatuse;
    4. Inimeste jagunemine kahte kategooriasse -
    · kurjategija;
    · mittekurjategija, on vastuolus terve mõistuse ja silmnähtavate faktidega. Kõik katsed moodustada teist gruppi on tulutud kuna mittekurjategijate hulgas on alati ka kurjategijaid. Kõik isikud on toime pannud üht või teist laadi hälbeid, aga ainult osadele tahetakse (suudetakse) märk külge panna;
    5. Seaduse rikkujatest paljastatakse väheseid, samas võivad paljud olla süüdi ja mitte väiksemal määral;
    6. Kuna sanktsiooni rakendatakse inimese, mitte aga kuritegeliku käitumise suhtes, siis kurjategijat iseloomustavad jooned mõjutavad karistust ja selle tagajärge;
    Arvesse ei võeta ainult kuritegu, vaid ka kurjategijat iseloomustavaid andmeid.
    7. Sanktsioonid sõltuvad kuuluvusest vähemusse, kindla elukoha olemasolust, haridusest, elukohast jne.;
    8. Õigusemõistmine lähtub stereotüüpsest ettekujutusest kurjategijast, kui madala moraali ja paha sooviga hukkamõistu väärivast isikust;
    9. Kui isik on kord märgistatud, siis on tal sellest märgist raske vabaneda .
    On tehtud uuringuid, mis on näidanud, et tagasipöördumine ühiskonda on raskendatud. Põhjuseks ei pea loomulikult olema ainult märk vaid selleks võib olla ka isik ise. Puhast eksperimenti märgi suhtes on raske teha juba eetiliselt ning samuti peaks eksperimenteeritav ise olema veendunud, et märk on õieti pandud.
    Lemert hakkas rõhutama, et märgistamise puhul on subjektiivne omaksvõtmine väga oluline. Eristab primaarset ja sekundaarset hälvet. Primaarse hälbe puhul paneb isik toime negatiivseid tegusid muudel põhjustel kui märk. Sekundaarse hälbe puhul on isik omaks võtnud kurjategija märgi ja paneb seetõttu toime negatiivseid tegusid.
    Lemert'i järgi on märgi omaksvõtt protsess, mis koosneb faasidest:
    1. primaarse hälbe toimepanek (selle põhjust ja olemust ei käsitleta);
    2. sanktsioon st. riik märkas hälvet ja sanktsioneerib. Algul on see kergem ja märgistamist ei toimu;
    3. järgnev primaarne hälve (üldjuhul ei pane isik seda enam peale esimest toime);
    4. raskem sanktsioon ja võõrandumine st. ühiskond hakkab isikusse kõrvalehoidvalt suhtuma kuigi märki veel ei panda;
    5. järgmine primaarne hälve koos võimaliku vaenulikkusega nende suhtes, kes karistavad;
    6. ühiskonna poolse kannatuse katkemine ja hälviku märgistamine. Tõstatub küsimus, et millise konkreetse teoga märk kinnistatakse? Erinevatel rühmadel on selleks erinevad meetodid;
    7. hälviku poolt hälviku rolli omaksvõtmine ja kohandumine lähtudes antud hälviku rollist. Alguses isik ei suhtunud endasse kui hälvikusse, aga nüüd suhtub. Sisemisest märgist vabanemine on märksa raskem kui välisest.
    Teatud märgistamine toimub alati. Märk omab toimet, sh. nii väline kui ka sisemine märk. Märgistamisel tuleb arvesse võtta kriminaalpoliitikat, et sellega mitte liialdada ja näha ette märgistamise tagajärgi. Positiivne oleks kui märgistataks vähem kui meil praegu.
    3.5. Subkultuuri teooria – R. Maruste raamat selle kohta
    Tuntuim esindaja on Cohen . Teooria vaatleb ühte rohkem isoleeritud kultuurinormide süsteemi ja nimelt negatiivset st. kuritegelikku subkultuuri (see teooria keskendub rohkem sellele, mida vastastikuse mõjutamise protsessis omandatakse). Teooria on seisukohal, et kurjategijatele on omane oma kultuurinormide süsteem, mis on erinev normikuulekate omast. See tähendab, et hälviku maailm on isiku maailm, kes allub seadustele , kuid ainult seadustele, mis on peapeale pööratud ja mille normid moodustavad jõu, mis on suunatud kehtiva korra vastu. A. Cohen vaatleb hälbiva subkultuuri arengu protsessi kui keskklassi väärtustele vastandlike käitumisnormide koodeksi tekkimist, toetamist ja tugevdamist. A. Coheni lähenemisel alaealiste kuritegevusele kui konkureerivate vastandlike väärtuste süsteemide tagajärjele on siiski tõsiseid puudujääke. Uuringutel on saadud huvitavaid tulemusi.
    Teatud ühiskonna lõike poolt on omaks võetud normide süsteem - nn. kuritegelik subkultuur, mis hõlmab peaaegu kõike. Kuritegelik subkultuur omab erinevate kurjategijate kihtides erinevat jõudu ja tähtsust.
    Selles teoorias (hälvik suhtub opma hälbivasse käitumisse kui õigesse) on paar probleemi:
  • Kui tõepoolest eksisteerib hälbiv subkultuur, mille järgi hälvik vaatleb oma ebaseaduslikku käitumist kui moraalselt õiget, siis oleks täielik alus oodata, et kinnivõtmisel ei oleks tal häbi- ega süütunnet  peaks olema nördinud, märterlik. Osa nii käituvadki, kuid see võib tuleneda kahjutundest, et vahele jäädi.
  • Uurimused näitavad, et alaealine õigusrikkuja väljendab sageli seaduskuuleka kodaniku suhtes austust ja vaimustust. Tõeliselt ausat inimest hinnatakse nende poolt sageli kõrgelt. Ja kui hälvik näeb kergesti läbi ning ei talu konformiste, siis äraostmata ausus kutsub temas üsna tõenäoliselt esile heakskiidu. Võib jätta tähele panemata, et nii alaealised kui ka täiskasvanud kurjategijad suhtuvad suure kiindmusega tagasihoidlikku, jumalakartlikku emasse või ausasse vaimulikke ning seletada seda sentimentaalsusega, kuid ilma kahtluseta on selge, et hälvik ei pea sugugi mitte igat seaduskuulekat isikut amoraalseks subjektiks. Samuti võib hälvik väljendada siirast nördimust, kui ebaseaduslikus tegevuses on süüdi keegi silmapaistvatest isikutest tema oma ringkonnast või mõni spordi- või estraadikuulsus.
  • On olemas suur hulk tõendeid selle kohta, et hälvikud teevad sageli ranget vahet isikutel, kes võivad saada nende ohvriteks ja isikutel, kes ei saa. Arvatakse, et teatud sotsiaalsete gruppide suhtes on mõned hälbivad teod keelatud, teiste suhtes lubatud. Hälbiva käitumise tõenäosus on võrdeline alaealise õiguserikkuja ja ohvri vahelise sotsiaalse kaugusega  hälvikud tunnetavad oma teo ebaõigsust märksa rohkem, kui seda tunnistatakse kirjanduses. Kui ohvrite ringi piiravad sellised tunnused nagu sugulus , etniline grupp, sotsiaalne klass, vanus, sugu jne, siis on alust arvata, et hälviku enda silmis pole tema käitumine sugugi laitmatu.
  • On kahtlane, kas tõepoolest enamik alaealisi õigusrikkujaid suhtuvad täiesti eitavalt nõudmisse alluda seadustele, mis lähtuvad domineerivast sotsiaalsest korrast. Üsna tõenäoliselt on alaealise õigusrikkuja perekond, isegi siis, kui ta ise tegeleb mingi ebaseadusliku tegevusega , respektaabli ühiskonnaga nõus selles, et hälbiv käitumine on negatiivne nähtus.
    Paljud uurijad on küüniliselt väitnud, et alaealine õigusrikkuja võib tajuda karistamist kui märguannet mitte vahele jäära, kuid on sama suur või suurem tõenäosus, et laps võib üldtunnustatud normid omaks võtta. See ei tähenda seda, et ei eksisteeri neid norme järgimast takistavaid tegureid. Hälbiva käitumise olemuse mõistmiseks on otsustava tähtsusega mõista, kuidas toimub väliste ja sisemiste käitumisnormide neutraliseerimine. Kui sügavalt ka ei oleks alaealine hälbivasse käitumisse sisse tõmmatud ja ükskõik mil määral sellesse kuulumine ka ei oleks talle tähtsam seostest seaduskuulekate isikutega, ikkagi ei saa ta hoiduda oma tegevusse suhtumast taunivalt. Kuid üldtunnustatud käitumisnormidele peab olema mingi vastukaal, alaealine ei saa lihtsalt neid ignoreerida ja suhtuda neisse kui võõrasse väärtuste ja normide süsteemi.
     Alaealine õigusrikkuja on vähemalt osaliselt seotud valitseva sotsiaalse korraga ja seetõttu kehtivaid norme rikkudees väljendab ta sageli häbi ja süütunnet, suhtub heakskiitvalt inimestesse, kes järgivad üldtunnustatud käitumiseeskirju ja teeb ranget vahet inimeste vahel, kes võivad tema puhul olla hälbiva käitumise ohvriks ja kes mitte.
    Kui pöörata tähelepanu neutraliseerimisteooriale ja võrrelda seda antud teooriaga, siis näib, et Cohen väidab vastupidist. Vastuolu on võimalik vaadelda:
    1. ühelt poolt sisulise vastuoluna. Sisulist vastuolu võib kõrvaldada sellega, et viidata erinevatele kurjategijate kihtidele;
    2. teiselt poolt näilise vastuoluna. Võib neutraliseerimisteooriad viia üheks subkultuuri elemendiks .
    Samas tuleks tähelepanu pöörata küsimusele kas esmakordne kurjategija on juba kuritegeliku subkultuuri kandja? Sellele küsimusele vastuse otsimisel tuleb "ettevaatlik" olla. Subkultuuri teooria kirjeldab paremini regulaarselt kuritegusid toimepanevaid isikuid ja neutraliseerimise teooria paremini ühekordse deliktse käitumisega või kuritegelikku subkultuuri mittekuuluva isiku käitumist.
    3.6. Kontrolli teooria.
    Esindajaks on Travis Hirschi. Siiamaani on kõik käsitletud teooriad lähtunud seisukohast, et me kõik oleme head. See teooria lähtub küsimusest, et miks me kogu aeg kuritegusid toime ei pane. Teooria rõhutab seda, et selleks, et isik kuritegusid toime ei paneks, peab olema teatud kontrollimehhanism.
    Kontroll jaguneb:
    1. väline kontroll;
    2. sisemine kontroll.
    Väline kontroll kujutab endast formaalsete institutsioonide tegevust (politsei, kool jne.). Välise kontrolli alla kuulub ka mitteformaalne kontroll (perekond, sõpruskond jne). Formaalne väline kontroll on üldiselt kuritegevuse vastase suunitlusega. Mitteformaalse kontrolli juures esineb ääretult harva ka juhte, kus need ei oleks kuritegevuse vastased. Mitteformaalsed grupid va. perekond on üldjuhul neutraalsed.
    Antud teooria on seisukohal, et välisest kontrollist ei piisa ja märksa olulisem on sisemine kontroll.
    Eduka sisemise ja välise kontrolli olemasolu tingimused:
    1. sidemed erinevate normikuulekate struktuuridega  mida rohkem, seda parem
    2. isiku pühendumine normikuulekate eesmärkide saavutamisele (haridus, erialane karjäär jne.)
    3. aja kulutamine konventsionaalseks tegevuseks, sealhulgas eesmärkide saavutamiseks normikuulekal teel; olulist tähtsust omab siin ka isiku intelligents (isik ei näe oma vaeva resultaati kaugemas perspektiivis kui ta on vähem intelligentne ja seetõttu ei vali normikuulekaid tegevusi, mille resultaat ilmneb alles kaugemas perspektiivis)  intelligentsuse küsimus
    4. normide järgimise mõttekusse uskumine positiivne üldpreventsioon (a la vaadake kui mõistlik ja hea on maailm, kui kõik järgivad norme).
    Sotsioloogilised makrotasandi teooriad kriminoloogias – ühiskonna kui terviku kaudu
    4.1. Adolphe Quetelet
    9.juulil 1831 esines Brüsselis Belgia Kuningliku Teaduste Akadeemia istungil kriminoloogia ajalukku läinud kõnega kuulus astronoom ja matemaatik A. Quetelet. Ta ütles: "Peaaegu samuti nagu me võime varem kokku lugeda, kui palju inimesi sünnib ja kui palju sureb, võime me eelnevalt välja arvutada, kui palju indiviide kõrvetab käed oma kaaskodanike veres, kui paljudest inimestest saavad kelmid, kui paljudest mürgitajad... Meil on siin tegemist arvega, mille järgi me maksame kohkumapaneva järjekindlusega - me maksame vanglate ja võllastega." See suurepärane järeldus sai võimalikuks tänu statistika arengule.
    Kuni XIX sajandini puudusid süstemaatilised statistilised andmed kuritegevuse kohta. Arvatavasti avaldas esimese maana kohtustatistikat Prantsusmaa. 1827 aastal ilmus seal esimene "Üldine aruanne kohtu jurisdiktsiooni täideviimisest Prantsusmaal".
    Quetelet (1788 - 1874 ) on üks sotsioloogilise koolkonna esindajaid. Tema kuulsaim raamat on 1835. aastal ilmunud “Sotsiaalne füüsika”. Hakkas tegelema kuritegevuse statistika uurimisega. Oli astronoom ja matemaatik ja seetõttu tegeleski kuritegevuse statistikaga.
    Kuritegevuse statistika hakkas kiiremini arenema 18. - 19. sajandi vahetusel ja Quetelet hakkas uurima, kas selle põhjal saab teha järeldusi. Püüdis leida ühiskonnast seaduspärasusi, mida oli varem leitud füüsikas. Jõudis veendumusele, et kuritegevus on püsiv nähtus ja seda on võimalik ennustada järgmiste aastate osas kuna ühiskond sisaldab kõigi tulevikus toimepandavate kuritegude eoseid, sest ühiskond valmistab kurjategijaid ette.
    Quetelet leidis rea statistilisi seoseid ja erisusi meeste ja naiste kuritegevuse vahel. Leidis palju seoseid ka sotsiaalsete tegurite muutuste ja kuritegevuse vahel. Nt kui tõusevad hinnad , siis tõuseb ka kuritegevuse tase.
    4.2. Emile Durkheim (1858-1917)
    Miks otsida põhjuseid ühiskonna tasemel? Ühiskond mõjutab neid. Kas sellele on empiirilisi aluseid? Erinevatel perioodidel on kiired muudatused kuritegevuses, mida ei saa mikrotasemel seletada.
    Ühiskonna eesmärk on koostööd tagada.
    Kuna Durkheim oli sotsioloog , siis kuritegevusega tegelemine ei olnud talle otseseks huviks. Vaatles enesetappe ja jõudis seisukohale, et mehi on enesetapjate hulgas rohkem kui naisi, üksikuid on rohkem kui perekonnainimesi. Suurem osa enesetappe leiab aset suvel linnas. Jõudis järeldusele, et enesetapud on seoses sotsiaalsete kontaktide nõrkusega. Durkheim tegutses selle sajandi aluses ja seetõttu ei pruugi kõik tema teooriad praegu paika pidada. Eestis tundub hetkel olevat maal enesetappe on rohkem.
    Ta püstitas 19. sajandi lõpul teesi, et kuritegevus on normaalne ühiskondlik nähtus. Inimeste kollektiivset teadvust ei ole võimalik sedavõrd unifitseerida, et nad alluksid kõigile sotsiaalsetele normidele. Kuritegevus ei ole mitte ainult paratamatu, vaid ka täiesti normaalne ja koguni soovitatav, kuigi kuritegevus on kultuuriti erinev.
    Ei ole ühtegi teist nähtst, mis oleks nii normaalne. Ta on tihedalt seotud elutingimustega. Kui oletada, et kuritegevus kujutab endast sotsiaalse patoloogia väljendusvormi, siis tähendab see seda, et patoloogia pole juhuslik, vaid vastupidi, teatud juhtudel ta kasvab välja elusorganismi põhistruktuuridestendist. Kuid sellega kaoks igasugune piir füsioloogilise ja patoloogilise vahel. Kui kuritegevus ületab tavalise astme, võib ta olla ebanormaalne ja patoloogilise iseloomuga .
    Kui ühiskonna arenemisega madalamatelt vormidelt kõrgematelekuritegevue tase ilmutaks kahanemistendentsi, siis võiks oletada, et kuritegevus, jäädes küll normaalseks nähtuseks, oleks järk-järgult taolist normaalset iseloomu kaotamas. Statistika näitab, et kuritegevus on kõikjal kasvanud.
    Pidades kuritegevust normaalseks, ei väida ta siiski, et see tuleneb inimeste pahelisusest. Kuritegevus on üks ühiskonna tervise faktor, lahutamata osa. Ühiskond ilma kuritegevuseta ei saa eksisteerida: ühiskonnas, kus kuritegusid enam toime ei panda, peaksid tunded, mida võiksid haavata kuriteod, olema omased kõigi liikmete individuaalsele teadvusele. Isegi kui see oleks täidetud, siis kuritegevus ei kaoks, vaid muudaks vormi, sest seesama põhjus, mis kaotab mainitud viisil kuritegevuse senised allikad, loob ise uued.
    Selleks, et seadusega kaitstud kollektiivsed tunded haaraksid täielikult ühiskondliku teadvuse või selleks, et need tunded omandaksid suurema jõu seal, kus nad on ebapiisavad, peavad need tunded muutuma intensiivsemaks kui varem. Ühiskond tervikuna peab kogema neid tundeid pingelisemalt, sest tal ei ole mujalt võtta jõudu, et kontrollida isikuid, kes varem kõige vähem on allunud nende tunnetele. Vastumeelsus tapmisele peab tekkima nendes kihtides, kust tapjad pärit on. kogu ühiskond võiks võtta kuritegevuse suhtes negatiivse hoiaku, sest mida rohkem on negatiivset hoiakud, seda rohkem see levib. Selline ühiskond on aga pea-aegu võimatu, kui ei tugevne need nõrgad tunded, millele kahju tekitamine oli varem vaid tava rikkumine. Nõrgemad tunded ei ole midagi muud kui tugevamate tunnete jätk, nende summutatum vorm (nt halb maitse vs vargus). Kuid kui see tunne tugevneb niivõrd, et sunnib vaikima vargusele õhutavad motiivid ühiskonna kõigi liikmete teadvuses, siis muutub see tunne tundlikumaks selliste rikkumiste suhtes, mis teda seni ainult kergelt puudutasid. Inimesed hakkavad niisugustele rikkumistele energilisemalt reageerima , neid hakatakse pidama häbiväärsemaks. See kõik viib selleni, et mõned taolistest rikkumistest lähevad moraalinormide rikkumiste ringist üle kuritegude hulka. Näide: pühakute ühiskonnas on vaadatakse nt ropendamisele kui kuriteole. NB! See sõltub keskkonnast.
    Samas ka ressurss liigub.
    Miks isegi kõige nõrgem tunne ei võiks olla piisavalt tugev, et vältida igasugust mitteallumist? Ühiskonna moraaliteadvus peab täielikult sisalduma ühiskonna iga liikme teadvuses ja olema piisava mõjujõuga, et vältida kõiki teda ohustavaid ründeid. See on võimatu, sest igaüht meist ümbritseb füüsiline keskkond, pärilikud omadused ja sotsiaalsed faktorid, mis mõjutavad kõiki erinevalt.
    Kuna ei saa olla ühiskonda, milles indiviidid ei erineks suuremal või vähemalt määral keskmisest kollektiivsest tüübist, siis järelikult on vältimatu, et selliste hälvete hulgas on ka kuritegelikke hälbeid. Oma iseloomu saavad need mitte teole sisemiselt omaste tunnuste järgi, vaid kollektiivse hunnangu järgi.
     Kuritegevus on vajalik, sest ta on tugevasti seotud igasuguse sotsiaalse eluga ja just seetõttu kasulik, sest need tingimused, mille osaks on kuritegevus, on ise lahutamatud moraali ja õiguse normaalsest evolutsioonist.
    Õigus ja moraal muutuvad ühiskonna sotsiaalse tüübi muutumisel ja et nad evolutsioneeruvad ka ühe ja sama ühiskonna tüübi raamides vastavalt selle ühiskonna elutingimuste muutumisele. Et need oleksid võimalikud, ei tohi moraali aluseks olevad kollektiivsed tunded suhtuda vanulikult muutustesse ja seega peavad nad olema mõõduka mõjujõuga. Kui nad on liiga tugevad, kaotab ühiskond paindlikkuse. Iga olemasolev süsteem on niivõrd, kuivõrd ta on kaotanud paindlikkuse, uue süsteemi arengu takistuseks. Mida täiuslikum on süsteem, seda suuremat, tervet vastupanu osutab ta muutustele. Kui ei oleks kuritegevust, tähendaks see seda, et vastupanu igasugustele muutustele on absoluutne. See eldaks, et kollektiivsete tunnete intensiivsus on enneolematult kõrge, kuid miski ei ole hea piiratult ja lõputult. Moraaliteadvuse jõud ei tohi olla liiga suur, sest muidu ei julge keegi seda kritiseerida ja ta võtab kergesti tardunud vormi. Ka hälbeid ei tohi olla liiga palju.
    Progressiks on vaja, et individuaalsus saaks väljenduda nii idealisti kui kurjategija puhul.
    Lisaks etendab kuritegevus ka olulist rolli ühiskonna evolutsioonis . Ta valmistab muutusi ette. Ühiskonnas, kus on olemas kuriteod, on kollektiivsedtunded piisavalt elastsed, võtmaks uut kuju ja kuritegu võib mõnel juhul määrata, millise vormi need võtavad ( Sokrates ja vaba mõtlemine – aitas kaasa usu ja moraali tugevnemisele; praegu ei saaks vabamõtlemist olla, kui Sokratese ajast poleks seda tehtud ja hiljem see lubatuks kuulutatud; liberaalsuse eelkäijateks olid ketserid ). NT: kloonimine (kuritegu, aga võib hakata ühiskonna hüvanguks töötama) – oh neid omapäraseid genotüüpe, samas klooonitakse inimeselaadseid olendeid muidu ja selle ohutus pole kindel, see vähendab ka geneetilist mitmekesisust.
    Seega, kui kuritegevus langeks alla keskmise, siis võib olla kindel, et selline näilik progress on seotud teatud sotsiaalse desorganisatsiooniga. Kuigi kuritegevus on normaalne nähtus, ei tulene veel sellest, et me ei peaks temasse negatiivselt suhtuma. Siiski, kuna kuritegu pole haigus, ei pea ka karistusse suhtuma kui ravimisse.
    Kuritegevuse põhjuste kirjeldamisel tõi sisse uue mõiste anoomia . Durkheim defineeris anoomiat kui ühiskonna seisundit, mida iseloomustab sotsiaalse tasakaalu puudumine ehk sotsiaalne organiseerimatus ning see on kuritegevuse peamine põhjus. Kõik muutused ühiskonnas tekitavad desorganiseeritust. Siis pole täielikku allumist normidele.
    See teooria on lähedane kontrolliteooriale (normide mitterikkumiseks on vajalik kontroll). Durkheim vaatab ainult välist kontrolli, sisest ei vaata.
    Kas anoomia on stabiilne seisund? Millest ta sõltub? Ühiskonna üldistest muutustest. Kas Durkheim seob anoomia tekke ainult siis negatiivsete nähtustega? Mõlematega.
    Miks anoomia tekib muutuste ajal? Ühiskond ei suuda norme piisavalt kiiresti omandada. Ühiskond peab hakkama viima uusi normi indiviidina, mis aga võtab kaua aega.
    Peale mõtte desorganiseeritusest tuli ta järeldusele, et kuritegevusel on oma funktsioon. (vt. ülevalpool esitatud teesi). Mis juhtuks, kui õnnestuks kuritegevus likvideerida ? Durkheim jõudis järeldusele, et sel juhul tunnistataks selline tegu, mis varem oli lubatud, kuriteoks. Selleks, et poleks enam ühtegi kuritegu, peavad kõik ühiskonna liikmed käituma väga ühtemoodi. Ei oleks enam hälbeid, mida kuriteoks tunnistada. Tegi lõppjärelduse, et selleks, et ühiskonnas oleks areng, on vaja ka kuritegevust. Kuritegevus on ühiskonna tervise tunnus st. ka terve ühiskond omab kuritegevust. Kuritegevus on normaalne nähtus ühiskonnas.
    Antud tees on vaadeldav kuritegevuse kui massnähtuse suhtes. Kuriteo hindamiseks positiivseks on vajalik ajaline distants . Durkheim ei olnud arvamusel, et kuritegevus on väga hea nähtus ja mida rohkem seda on seda parem. Ta leidis, et ühiskonnas peab olema normaalne kuritegevuse tase. (Hetkel ei ole Eestis ohtu, et me võiksime seada ohtu kuritegevuse vastu võitlemisel ühiskonna arengu.) Durkheim arvas, et üksikkuritegu on negatiivne ja selle vastu peab võitlema. Peame tunnistama, et nende hulgas võib olla positiivseid, kuid see ei saa takistada kuritegevuse vastast võitlemist.
    Kuritegevuse teise funktsioonina nägi Durkheim seda, et kuritegevuse vastu võitlemine aitab kaasa (kuritegevusevastasel võitlusel tekib) ühiskondliku ühtekuuluvustunde suurenemisele. Näide: aprillirahutusteaegne ühtekuuluvustunne, naabrivalveprojektid  teatud suurenemist näeb, aga ei midagi sellist väga piiri mööda jooksvat (kõik normikuulekad on kõigi normirikkujate vastu). Välisele rünnakule reageerib normikuulekas ühiskonna osa ühtse tegevusega. Kui normikuulekas ühiskond on suurem, kui kuritegelik, siis peab või võib see kõik paika pidada. Kõrge kuritegevuse korral normikuulekat ühiskonda ei teki. Ei ole selget eraldatust, piirid hägustuvad.
    Kuritegevus tekib, sest muutused ühiskonnas tekivad ja muutused on võimalikud tänu teatud kuritegevuse tasemele .
    NÄITED:
  • Eesti 1989-1992
  • Eesti – eraomand
  • Iraakmuudatuste edukuseks on vaja reeglite muutust ja ühiskonnaliikmeteni viimist
  • Kultuurikonflikti näited sobivad ka anoomia allikateks.
  • Lühiajalised muudatused – New Orleans (ka kontrolliteooria juurde), natuke ka aprillisündmused (seal oli muud ka, nt massipsühhoosi) – mis reeglite indiviidini viimisega raskusi tekkisid? Anoomia ühiskonna reeglite indiviidini viimise mehhanismides. Seal hakati jagama ka toitu ja joogivett jne, aga selle reegleid ei suudetud indiviidini viia, samuti varem kehtestatud reeglite mehhanismid lõigati läbi.
  • Kui ühiskond ühel hetkel keelustab abordi.
    4.3. Robert Merton, eesmärkide ja vahendite konflikt kuritegevuse
    põhjusena.
    Robert Merton oli samuti sotsioloog. Soovis jääda termini anoomia juurde. Pidas anoomia tekkepõhjusi teistsugusteks. Anoomia tekib siis, kui on olulised vastuolud sotsiaalses struktuuris. Vaatles kuritegevuse põhjusena eesmärkide ja vahendite konflikti.
    Sotsiaalne struktuur:
    · Eesmärgid – pürgimused; nad on rohekm või vähem integreeritud, põhiline komponent „grupilise eksisteerimise skeemist“. Need võivad olla seotud ürgsete tungidega, kuid pole selle poolt ette määratud. Samas tahame ka kuuluda gruppi ja sotsiaalselt ollakse nii arenenud ja arenetakse ka kiiremini (see on geneetiliselt sisse kodeeritud). Tunnustus aitab kaasa koostööle.
    · Vahendid – iga sotsiaalne grupp seob soovitavate eesmärkide hierarhia nende eesmärkide saavutamisel lubatud ja nõutud vahendite formaalse või mittevormaalse reguleerimisega. Vahendite lubatavus ja selliste regulatiivsete normdie ja moraalsete imperatiividejärgi ei lange sugugi alati kokku mainitud vahendite tehniliste otstarbekuse ja efektiivsusega. Paljud vahendid, mis tunduvad üksikisikule kõige efektiivsemad (nt ebaseaduslikud tehingud naftakompaniide aktsiatega , vargus, pettus), on väljaspool lubatud käitumise sfääri.
    Miks väide, et kuritegevuse põhjus on legaalsete efektiivsete vahendite vähesus, ei ole korrektne ? Seal ei ole enam juttu eesmärkidest. Kui vaadelda ainult vahendeid, siis on neid tegelikult tunduvalt rohkem kui aastatel nt 1600-midagi jne.
    Ühiskond määrab ära ükskonna liikmete eesmärgid ja lubatud vahendid nende eesmärkide saavutamiseks. Nende vahekord ei pea olema püsiv, nt mõnede eesmärkide tähtsus võib muutuda sõltuvalt lubatud vahendite tähtsuse astmest. Teatud juhtudel võib esineda ebaproportsionaalne, vahel isegi praktiliselt absoluutne teatud eesmärkide rõhutamine ja nende eesmärkide saavutamise vahendite lubatavus jääb tähelepanu alt välja. Sellise situatsiooni äärmuslikuks väljenduseks on juhtum, kus valik alternatiivsete vaendite vahel toimub ainult tehnilistest, mitte aga sotsiaalsetest normidest lähtuvalt. Sellises hüpoteetilises äärmuslikus situatsiooonis oleks lubatavad kõik vahendid, mis suudaksid tagada eesmärgi saavutamise. Teine äärmuslik variant ilmneb nendes gruppides, kus tegevus, mis algselt oli mõeldud eesmärgi saavutamiseks, muutub ise eesmärgiks. Sellistes gruppides esialgsed eesmärgid ununevad ja äärmusliku tähtsuse omandavad ettekirjutatud vahendid ja nende kasutamine. Pööratakse suurt tähelepanu stabiilsuse tagamisele ning muutustesse suhtutakse samal ajal halvakspanuga. Alternatiivsete käitumisvariantide valik on rangelt piiratud. Areneb välja traditsioonidega seotud püha ühendus, mida iseloomustab neofoobia. Näiteks võiks siin olla bürokraadi ametialane psühhoos. Muidugi on olemas ka vahepealset tüüpi grupid, milles kultuuri poolt määratletud eesmärkide ja lubatud vahendite vahel valitseb tasakaal. Selliseid gruppe iseloomustab tugev integratsioon ja suhteline stabiilsus, mis aga ei takista muutusi.
    Mainitud kahe sotsiaalse struktuuri vahel on võimalik efektiivne tasakaal vaid seni, kuni isikud, kes alluvad mõlematele piirangutele, saavutavad rahulolu (jutt on rahulolust eesmärkide saavutamise üle ja rahulolust ühiskondlikult heakskiidetud vahendite kasutamise üle). Sellistel tasakaalustatud juhtudel koosneb edu kahest momendist. Edu hinnatakse siin resultaadi ja protsessi terminites e lõpptulemuse ja tegevuse terminites. Püsiv rahuldus peab tulenema konkurentsis osalemisest endast ja konkurendi ees edu saavutamisest. Juhuslikud ohvid, mis seonduvad ühiskondlikult heakskiidetud käitumisega, peavad olema sotsiaalsete stiimulitega kompenseeritud. Staatuste ja rollide jaotus võistluse kaudu peab olema organiseeritud nii, et iga ettenähtud positsiooni jaoks oleks mingi positiivne faktor, mis stimuleeriks igaühe vastavust oma rollile ja staatusega seotud kohustuste täitmist. Seega hälbivat käitumist võib vaadelda kui kultuuri poolt määratletud pürgimuste ja ühiskondlikult heakskiidetud vahendite kooskõlastamatuse sümptomit.
    Vahendid jaotuvad:
    · efektiivsed vahendid;
    · ebaefektiivsed vahendid.
    või:
    · legaalsed vahendid;
    · illegaalsed vahendid.
    Teooria seisukohalt on huvitavad need grupid, kus eesmärkidel on ebaproportsionaalselt suur rõhk. Pole olemas selliseid gruppe, kellel puuduksid käitumist reguleerivad normid. Erisused on aga selles, kuivõrd seotud on tavad, harjumused ja formaalne kontroll eesmärkidega. Emotsionaalsed suhtumused võivad grupeeruda ühiskondlikult tunnustatud eesmärkide ümber ning jätta oma toetusest ilma antud kultuuri poolt määratletud eesmärkide saavutamise vahendid. Nagu allpool näeme, võivad mõned sotsiaalse struktuuri aspektid kutsuda esile moraalivaese või antisotsiaalse käitumise just nimelt sellega, et eesmärkidele ja nende saavutamise vahenditele antakse erinev tähendus. Äärmuslikel juhtudel võib eesmärkide ülemäärane rõhutamine tingida vahendite valiku ainult lähtuvalt tehnilisest otstarbekusest . Merton jõudis järeldusele, et efektiivsed vahendid on sageli illegaalsed (nt sport jt grupid, kus puudub sotsiaalse struktuuri kahe elemendi vaheline piisav seos ja ühiskonna rikkuse kogumisele kui edu sümbolile osutatav ülim tähtsus takistab efektiivset kontrolli varanduse hankimise vahendite seaduslikkuse üle, nt korruptsioon , amoraalsus) ja legaalsed vahendid on sageli ebaefektiivsed. Püüd eesmärgi saavutamisele vähendab sel juhul saadavat rahulolu sedavõrd, et see rauldus muutub ainult rahuloluks saavutatud tulemuse üle. Väikesed südametunnistuse piinad ja reeglite rikkumise salajane iseloom näitab selgelt, et ettenähtud reeglid on rikkujale teada, aga reeglite emotsionaalsed toed on oluliselt nõrgendatud eesmärgile osutatava tähtsuse ülehindamisega kultuuri poolt.
    Kui vaadelda erinevatele ühiskondadele omaste erinevate sotsiaalselt hälbiva käitumise tüüpide sotsiaalset geneesi, siis peab vaatlema ka teisi aspekte. Lähtuvalt 3st sotsiaalse korra ideaalsest tüübist võib eristada viit loogilist võimalikku, alternatiivset infiviidi ühiskondlike tingimustega adapteerumise viisi.
    Isikud võivad ühest alternatiivist teise sõltuvalt sellest, millise sotsiaalse tegevuse liigiga nad parasjagu tegelevad. Toodud kategooriad kirjeldavad isiksuse adaptsiooni konkreetsesse rolli spetsifilises situatsioonis, mitet aga isiksust tervikuna. Oma töös vaatleb Merton eelkõige majanduslikku tegevust (tootmist, vahetust, tarbimist jne).
  • Allimine - Igas ühiskonnas on kõige tavalisem allumine (tagab stabiilsuse ja järjepidevuse; lubab rääkida kogumist kui ühiskonnast).
  • Innovatsioon (majanduslik) – ebaõnnestumisega seotud konflikt ja üleelamised kõrvaldatakse loobumisega kasutatavate vahendite seaduslikkusest, tunnistades aga siiski edasi ühiskonna määratletud eesmärke.
  • Ritualism (majanduslik) – äärmiselt tugevalt omandatud piir lubatavate ja lubamatute vahendite vahel viib ritualismi mõnikord, mille puhul loobutakse eesmärgist kui võimatust, kuid jätkuvalt allutakse vahendeid määravatele ettekirjutustele.
  • Retritism - Kõige harvemini esineb retritismi, mis eitab eesmärke ja vahendeid. Inimesed sel juhul asuvad küll ühiskonnas, aga ei kuulu sinna (sotsioloogilises mõttes kontvõõrad – psühhopaadid, paariad, hulkurid, alkohoolikute ja narkomaanide tegevuse liigid). Seda tüüpi kohanemine toimub juhul, kui isik on täielikult omaks võtnud kultuuri poolt määratletud eesmärgid ja lubatavad vahendid ning ta on neid emotsionaalselt kõrgelt hinnanud, kuid efektiivsed lubatavad vahendid on talle kättesaamatud. Lüüasaamise meeleolud leiavad väljundi psühholoogilistes tegelikkusest põgenemise mehhanismides ning viivad vältimatult põgenemiseni ühiskonna nõudmiste eest. See on pideva seaduslike vahenditega eesmärgi saavutamise katsete nurjumise ning sisemise ja välise kontrolli tõttu ebaseaduslike vahendite kasutamise võimetuse tagajärg. Samal ajal ei ole selle protsessi käigus loobutud veel üldtunnustatud eesmärgist. Konflikt laheneb mõlemast elemendist loobumisega. Põgenemine on seega läbi, konflikt lahendatud , indiviid kohanenud ühiskonna nõudmistega.
  • Mäss (majanduslik) – üleminekureaktsioon, mis püüab kehtestada uued eesmärgid ja sotsiaalne kord, st alternatiiv on suunatud olemasoleva struktuuri muutmisele, mitte kohanemisele olemasoleva struktuuriga.
    Kriminoloogiat huvitab ebaseaduslik kohanemine, mis on seotud üldiselt keelatud, kuid efektiivsete vahendite kasutamisega kultuuri poolt määratud eesmärkide saavutamiseks. J. D. Lohmanni näide: ta tõestas, et eriline pahelisuse piirkond (mängupõrgute ja lõbumajade rajoon) ühes Chicago eeslinnas kujutab endast normaalset reaktsiooni situatsioonile , milles on omandatud kultuuri poolt määratletud rõhk rahalise edu saavutamisele, kuid on väga vähe üldtunnustatud ja seaduslikke vahendeid selle saavutamiseks. Kui arvestada põlgust, millega sealse kultuuri süsteemis suhtutakse füüsilisse töösse ja seejuures kõrget vaimse töö prestiiži, siis saab selgeks, et niisuguse situatsiooni tragajärjeks saab olla innovatsioon. Kvalifitseerimata tööaladega oma võimaluste piiramine ning sellest tingitud väike sissetulek ei suuda üldtunnustatud eesmärkide saavutamise mõttes konkureerida organiseeritud pahe ekspluateerimisega seotud suurte tuludega. Sellega on seotud 2 asjaolu:
  • Selline antisotsiaalne käitumine on teatud mõttes tingitud mõningatest üldtunnustatud eesmärkidest ja klassistruktuurist, millega kaasneb erinev vahendite kättesaadavus. Eesmärkide ja vahendite vahelise seose nõrkus ning klassistruktuur soodustavad sellistes gruppides antisotsiaalse käitumise suremat sagedust.
  • Allumise kasutamist piirab fakt, et üldtunnustatud eesmärkide poole liikumine üldtunnustatud teede kaudu (vastupidiselt saapapuhastajast miljonäriks ideoloogia kaitsmisele), on suhteliselt raske. Takistuseks on formaalse hariduse puudulikkus ning piiratud majanduslikud ressursid . Grupis eksisteerivate edustusstandardite domineeriv mõju viib seepärast seaduslike, kuid ikka ja jälle ebaefektiivsete esmärgi savutamise katsete väljatõrjumiseni ja üha sagedama ebaseaduslike, kuid enam-vähem efektiivsete amoraalsete või kuritegelike vahendite kasutamiseni. Kultuuri nõudmised isikule on sel juhul omavahel vastuolus. Ühelt poolt ta nõuab käitumise orienteerimisest varanduse kogumisele, kuid teiselt poolt ei paku peaaegu mingisuguseid võimalusi teha seda lubatavate vahenditega. Sellise struktuuri ebajärjekindluse tagajärjeks on psühhopaatilise isiksuse kujunemine ja antisotsiaalne käitumine ja revolutsiooniline käitumine. Kultuuri poolt määratletud eesmärkide ja vahendite vaheline tasakaal muutub järjest ebapüsivamaks vastavalt sellele, mida enam suureneb rõhuasetus eesmärgi saavutamisele iga hinna eest.
    Teisest asjaolust tuleneb see, et kui tahta mõista antisotsiaalse käitumise sotsiaalseid põhjuseid, peab eesmärkide kõrval arvestama ka teisi sotsiaalse struktuuri elemente. Paljud hälbiva käitumise juhud ei ole tingitud lihtsalt võimaluste puudumisest või rahalise edu ülemäärasest rõhutamisest.
    Antisotsiaalne käitumine omandab olulise mastaabi ainult siis, kui kultuuriväärtuste süsteem tõstab kõge kõrgemale teatud, kogu elanikkonnale ühised edu sümbolid, kuid samal ajal ühiskonna sotsiaalne struktuur piirab oluliselt või välistab täielikult suuremale osale elanikkonnast juurdepääsu seaduslikele ja efektiivsetele vahenditele nende sümbolite saavutamiseks. Eeldatakse, et need eesmärgid on klassierinevustest kõrgemal ja ei ole nende poolt piiratd, aga tegelikuses sotsiaalne struktuur tingib klassierisused edu sümbolite saavutatauvses. Ameerika ühiskonnale omane rahalisele edule ülemäärase tähtsuse omistamine ning kõigi hulgas auahnuse kultiveerimine tekitab seega ärevust, vaenulikkust, neuroose ning antisotsiaalse kätiumise sagenemist.
    Ainult niivõrd, kuivõrd vaesus ja sellega seonduvad raskused konkurentsivõitluses esinevad koos varanduse kogumisele kui edu sümbolile omistatava tähtsuse tajumisega, on antisotsiaalne käitumine vaesuse normaalne tagajärg. Seega on vaesus Kagu-Aasias märksa vähem seotud kuritegevusega kui Ameerikas. Vertikaalse sotsiaalse mobiilsuse võimalused on nendes maades väiksemad kui Ameerikas. Seega ei vaesus iseenesest ega sellega seonduv võimaluste piiratus ei võimalda seletada korrelatsioonide erisusi. Ainult sel juhul, kui me vaatleme kogu konfiguratsiooni, mis koosneb vaesusest ja võimaluste piiratusest, aga samuti kõigile ühistest edu sümbolitest koos, on võimalik seletada, miks USA-s on vaesuse ja kuritegevuse korrelatsioon tugevam kui teistes ühiskondades, kus range klassistruktuur seondub igale klassile erinevate edukuse sümbolitega.
    Sellise olukorra tekkimine ja sellega seotud käitumine on kultuuri süsteemis oleva koordineerituse puudulikkuse tagajärg.
    Kirjeldatud sotsiaalne protsess viib ühiskonna lagunemiseni. Sotsiaalse korralduse poolt avaldatav surve sunnib konkurente ennetama. Vahendite valik toimub lubatavate variantide vahel ainult niikaua, kuni konkurentsisüsteemi toetavad emotsioonid , st emotsioonid, mis tulenevad konkurendi edestamise ja sellega ümbritsevatelt positiivse reaktsiooni saamise võimalusest, hõlmavad kõiki tegevusvaldkondi. Sotsiaalse struktuuri stabiilsuse tagamiseks on vaja, et emotsioonid oleksid struktuuri elementide vahel ühtlaselt jagunenud. Kui võistluse protsessist rahuldust enam ei saada ja tähelepanu koondub üksnes edule, siis tekib pinge, mis blokeerib reguleerivad struktuurid. Sellistel juhtudel saavad vägivald ja pettus tähtsaimateks vahenditeks, kuna need on eesmärkide saavutamisel efektiivsed.
    Merton defineeris anoomiat kui seisundit, kus puuduvad efektiivsed ja legaalsed vahendid soovitud eesmärgi saavutamiseks (koordineerituse puudumine). Üheks üldisemaks sotsiaalse organisatsiooni funktsiooniks on aluse loomine inimeste käitumise prognoositavuseks ja reguleeritavuseks, kuid selle funktsiooni efektiivsus on seda piiratum, mida lahknevamad on mainitud sotsiaalse struktuuri elemendid. Äärmuslikel juhtudel prognoositavus kaob täiesti ja saabuvat olukorda võib nimetada kultuuri kaoseks või anoomiaks.
    ---
    Merton väitis aga hoopis, et kuritegevus ja hälbimine on modernse ühiskonna lahutamatu osa, s.t, et ühiskond koosneb kahest suhteliselt iseseisvast sfäärist: kultuurist ja sotsiaalsest struktuurist. Kultuur ütleb meile kahte asja: mida me peaksime soovima ja kuidas me peaksime käituma. Struktuur aga paneb paika selle, kuidas on jagatud võim, rikkus ja staatus. Traditsiooniliste ühiskondade struktuur on hierarhiline . Mõned inimesed on rikkad ja võimukad; enamus tähtsusetud ja vaesed. See vastuolu peegeldub ka kultuuris. Erinevaid inimgruppe õpetatakse elust väga erinevaid asju ootama ning käituma vastavalt oma seisusele. Seega on inimeste ootused tegelikkusega tasakaalus. Vaene inimene eeldab, et ta jääbki vaeseks. Seetõttu ta lepib oma vaesusega. Keskaegses Euroopas ja hinduistlikus Indias õigustati sellist ülimalt ebaõiglast elukorraldust religiooniga, mis lubas järgmises elus suurt tasu nendele, kes lepivad alandlikult oma positsiooniga selles elus. Allaheitlik kristlane loodab pärast surma “pärida maa”, kuid ta aga ei ürita seda selles elus endale krahmata. Vaene kuid väga hindu sünnib järgmises elus parematesse tingimustesse.
    Modernse sotsiaalse süsteemi muudab oma olemuselt konflikte õhutavaks asjaolu, et kultuur ja sotsiaalne struktuur pole enam omavahel kooskõlas. Kultuur on demokraatlik: kõigil on lubatud pürgida materiaalse heaolu poole. Ameerika unelm tähendabki seda, et igaüks võib saada USA presidendiks või vähemalt suurkorporatsiooni juhiks. A. Carnegie on öelnud: “Ole oma unistuses kuningas. Ütle iseendale : “Minu koht on tipus” .” Merton läheb veelgi kaugemale ning väidab, et Ameerika Ühendriikides (mis selles osas erineb teatud määral Euroopa ühiskondadest) peetakse edasipüüdlikkust lausa patriootiliseks.
    Kuid võrdsete püüdlustega ei kaasne võrdsed võimalused. Meritokraatlik retoorika ergutab kõiki inimesi endale samu eesmärke püstitama, kuid tegelikkuses ei võimalda klassistruktuur paljudel oma sihte legaalsete vahenditega saavutada. Kuna sotsiaalne struktuur ei võimalda neil ühteaegu jääda truuks ühiskonna poolt aktsepteeritavatele eesmärkidele ning ühiskonna poolt aktsepteeritavatele vaheniditele, kaotavad nad paratamatult usu ühte (või mõlemase) väärtuste süsteemi osasse.
    Merton leidis, et loeb "lõhe" ühiskonnas. Tavalähenemisest erineb Mertoni teooria sellepoolest, et rõhutab seda, et vahendite ebapiisavus on sageli suhteline. Ajaloolises mastaabis vaadates võib öelda, et vahendeid oli vähem ja nad olid raskemini kättesaadavad, kuid samas olid eesmärgid madalamad st. eesmärgid olid vahenditele lähemal. Oluline ongi see, et "lõhe", st vahemaa vahendi ja eesmärgi vahel määrab ära kuritegevuse. Oluline on see millised eesmärgid ühiskond inimese ette annab ja millised inimene endale ise endale sel juhul valib. Inimest häirib ainult see, kui suur on "lõhe" konkreetselt tema vahendite ja eesmärkide vahel. Konflikt eesmärkide ja vahendite vahel tekitab pingeid ja see omakorda kuritegevust.
    Majanduse seisukohalt on ühiskond edasipüüdlikum, juhul kui eesmärgid on ühiskonnas enam-vähem ühtsed, kuigi mõnele üksikindiviidile on need kättesaamatud. Majanduse seisukohalt loeb võimalus oma potentsiaali kasutada.
    Merton jõudis järeldusele, et see, mis võib olla kasulik kuritegevuse ärahoidmiseks, võib mõnele muule vajalikule valdkonnale (nagu nt majandus) olla kahjulik.
    Merton uuris ka inimese tegutsemist situatsioonis, kus (efektiivsed ja legaalsed) vahendid puuduvad:
    1. Ka suure konflikti jääb puhul inimene eesmärkide ja legaalsete vahendite juurde ning see on nn. konformne käitumine.
    2. Juhud, kui inimene algul jääb legaalsete vahendite ja eesmärkide juurde ning hiljem läheb üle illegaalsete vahendite juurde ning see on nn. innovatsioon. Innovaatoreid huvitab eelkõige lõpptulemus – reeglitest nad aga ei hooli. Pidev edukuse ülistamine, millega kaasnevad ebavõrdsed võimalused seda saavutada, võivad paljudes inimestes tekitada tunde, et nende püüd leida elus edasijõudmiseks uusi (ja ebaseaduslikke) viise on igati õigustatud. Noor itaallane, kes mõistab, et temast ei saa parimagi tahtmise juures General Motorsi presidenti, üritab selle asemel saada hoopis maffiaperekonna peaks.
    3. Juhud, kus inimene, nähes eesmärkide saavutamise võimatust, loobub eesmärgist, kuid ei loobu vahenditest ning see on nn. ritualism. Briti ühiskonnaajaloo- alane kirjandus lausa kubiseb uurimustest, milles käsitletakse madalama keskklassi esindajaid, kelle jaoks soov kuuluda “lugupeetud” inimeste hulka näib olevat muutunud peaaegu kinnisideeks; seda tüüpi inimesi on oma esimestes romaanides hästi kirjeldanud ka G. Orwell . Tegemist on inimestega, kel pole mingit suurt väljavaadet elus edu saavutada (ja kes suhtuvad liigsetesse ambitsioonidesse isegi kahtlustavalt: igaüks peab oma kohta elus teadma!), kuid kes kardavad sellegipoolest töölisklassi tagasi langeda. Rõivastus ja kõneviis on taoliste inimeste jaoks oluline vahend, mille abil tõmmata selge joon lugupeetud kodanike ja matside vahele. Merton ütles selle kohta järgnevat: “Selline maailmavaade iseloomustab ärahirmutatud töötajat, erapanga tellerit, kes kõigist reeglitest innukalt kinni peab, või riigiasutuse alumisel korrusel infoletis istujat”.
    4. Juhud, kus inimene loobub nii eesmärkidest kui vahenditest st. irdub teatud mõttes ühiskonnast ning see on nn. retritism. Nt hipid.
    5. Mäss, kus eitatakse nii eesmärke kui ka vahendeid ning püütakse kehtestada uut ühiskonna struktuuri st. uusi eesmärke ja vahendeid.
    Kõige levinum ühiskonnas on konformne käitumine. Kui see nii ei ole, siis on ühiskond väga haige. Innovatsioon on hälvetest kõige sagedasem ning võib öelda, et just see ongi kuritegelik käitumine. Ritualismi ja retritismi esineb tunduvalt harvemini - eriti viimast kuna raske on välja mõelda uusi eesmärke ja vahendeid.
    Kriminoloogia uurimismeetodid
    5.1. Kuritegevust iseloomustavad näitajad
    5.2. Latentne kuritegevus – vahe registreeritud ja varjatud kuritegevuse vahel
    Kõige selgem latentsuse põhjus: kurjategija ei tea, et ta on kuriteo toime pannud, ohver puudub või ohver ei tea, et tegemist on kuriteoga.
    Miks kannatanu jätab teatamata: kahju pole suur, kulub aega või kui suur kahju korvatud saaks. Seksuaalkuritegudest jäetakse teatamata sageli, nt prestiiž. Samas ei saa ka nt illegaalsed immigrandid politseisse lihtsalt pöörduda. Kardetakse ka kättemaksu. Need on latentsuse legaalsed vormid.
    Kuna politseid hinnatakse suuresti lahendamise statistika alusel, siis ei taha nad ka registreerida kuritegusid (nt kui näevad, et seda lahendada on liiga keeruline). Milliste parameetrite järgi peaks siis hindama ? Näiteks mitu minutit möödus politseile helistamise ja kohale jõudmise vahel. See on tugevalt seotud avastamise tõenäosusega.
    Kuidas latentseid kuritegusid uurida? Kui sellest ei teata, on see raske.
    Ei maksa arvata, et registreeritud kuritegevuse andmed peegeldavad tegelikku kuritegevuse seisu. Registratsioonist välja jääb sama palju kui mitte rohkem kui neid registreeritud saab. See vahe registreeritud kuritegude ja tegelike vahel ongi latentne kuritegevus – see, mis registratsioonist välja jääb. Registreeritud kuritegevuses on omaette ala ­– avastatud kuritegevus. Ei tohi panna, et latentne on see, mis jääb avastamata.
    Põhjused: kuritegu on nii hirmus asi, et peaks nagu saama registreeritud.
    • Kõige selgem ja automaatsem põhjus on need juhud, kus kurjategija ise ei tea, et ta kuriteo toime on pannud või ei ole kannatanut või kannatanu ka ei tea, et ta kuriteo läbi kannatada saanud on. Siia rubriiki väga palju kuritegusid ei lange. Pigem need olukorrad kus kannatanut ei ole, nt relva ilma loata omamine – isik ei pruugi olla keelunormist teadlik.
    • Kurjategija saab väga ilusti aru, et ta on kuriteo toime pannud, aga ei ole kannatanut – sama relva ebaseaduslik omamine. Teod jäävad üldreeglina latentseks, sest kurjategijal endal suurt huvi seda relva omamist üles tunnistada ja tahta karistamist, ei ole. Teatud etappidel võetakse hoiak, et olge head tulge oma ebaseaduslike relvadega, andke ära, aga me ei karista. Latentsus on seda tüüpi kuritegude puhul väga suur. Nt maksukuriteod – riigieelarvel jääb küll tulu saamata ja ühiskond seeläbi nagu kannatab.
    • Kannatanu on olemas, aga ta ei tea, et ta kannatanu on. Riigi poolt kannatanu olek maksukuritegudes. Selliseid, kus isik ei tea, neid on rohkem. Eriti siis, kui me võtame kuritegude kõrval ka väärteod. Nt pisivargused – spetsiifiline varaste kiht, kes püüavad nii, et omanikule ei jääks selget tunnet , et omanikult midagi varastatud on. Nt sendipealt ei tea, palju on raha rahakotis . Ehitustel kipub palju materjali kaduma minema, eriti sellist, mida on raskem jälgida.
    • Kannatanu on olemas ja ta ka teab ja ei teata. See on ainult teatav osa latentsest kuritegevusest!! Siin peab mõtlema selle peale, mille pärast kannatanu jätab teatamata – mis on põhjuseks. Siin on põhjuseid päris palju erinevaid:
      • Kui suur kahju on, kui palju aega läheb menetlemiseks ja kui suur on tõenäosus oma kahju korvatud saada – need toimivad koos ja kannatanu otsustab, kas teatamine on otstarbekohane oma aja kulutamise kohapealt – kui on väheväärtuslik asi, siis ega seda ilmselt ei avastata. – see osa on põhiline, miks kannatanu ei teata.
      • Kas vara on kindlustatud või mitte – niipea kui on kindlustatud, tõuseb %. Kindlustus maksab vaid siis, kui on teatatud.
      • On võimalus, et jätab teatamata, et ta ei taha kurjategija vastutusele võtmist – kui kannatanu isiklikult tunneb, nt perevägivald.
      • Kannatanu jätab teatamata enda prestiiži kaitseks – kardab , et kuriteo ohvrina teatavaks saamine mõjutaks teda negatiivselt. Nt seksuaalkuriteod – alateadvusest on palju hirmusid – sageli pole ka põhjendamatud. Siin on nt vaja selgitada, et ohvrid ei ole kuidagi selles kuriteos süüdi. Tuleb teha mingeid kampaaniaid. Aga ka mitmesugused juriidilised isikud, kellel ei ole mitte kasulik anda infot et nad kuriteo ohvriks on langenud, kõige rohkem ehk usaldusväärsus võib langeda – nt kui mõnel pangal osutub netipank ebaturvaliseks. Samuti kindlustusfirmad.
      • Ei teata, kuna ei taha politseipoolset tähelepanu. Illegaalsed immigrandid. Meil illegaalseid immigrante on oluliselt vähem kui mõnes muus arenenud riigis. Aga ka juhul kui isiku sissetulekuallikad on illegaalsed vms.
      • Kättemaksu kartus – isik ei teata kuna ta kardab, et kui teatab , siis kurjategija kohtleb teda veelgi negatiivsemalt – perevägivald.
      • On ka võimalus, et kannatanu teatab, aga politsei jätab kuriteo registreerimata. Pärast politseisse pöördumist tegu registrisse ei jõua. Ei taha teha ekstra tööd vms.

    Kui politsei tegevust hinnatakse liiga tugevasti selle järgi kui suur on kuritegude avastamise %, siis kipub samuti olema nii, et ei ole politseiametnikud sugugi mitte huvitatud, et registreerida selliseid, mille avastamine ei ole tõenäoline. Statistikas on huvitavaid hüppeid – hüpete eel oli just see, et pandi uus šeff – oli kergemalt valmis pingutama, et kõik toimepandud teod saavad registreeritud. Nt on parameetrid , mitu minutit möödub sellest, kus kannatanu helistab politseile ja see ka kohale jõuab – see on seotud ka sellega, kas kuritegu saab avastatud või mitte.
    5.2.1. "Self- report " uuringud
    Lastakse kirja panna kuriteod ja kui palju neid toime pandud on. See töötab väga piiratud situatsioonis (usalduslik vahekord – nt noortel). Enamusaldusväärsed on need andmed, kus on väiksed kuriteid. Enam kui 90% on toime pannud väiksemaid liike. Võib esineda ka valepositiivseid
    • Peab võtma päris hulga reservatsioone – on osa kuritegevusest, mida ühegi meetodiga ei anna välja selgitada – kui keegi ei tea, et kuriteo sündmus on toimunud – keegi ei saa kunagi head hinnangut anda.

    • See, kui kurjategija juba teab – praktiliselt kogu osa latentsest kuritegevusest – seda saab uurida Self-report metoodikaga – inimeste käest küsitakse – olge head, pange kirja kas ja kui palju te kuritegusid toime pannud olete – peab olema väga hea usalduslik vahekord küsijate ja vastajate vahel. Reaalses elus töötab see kolledži üliõpilaste seas. Samas peavad olema kindlad, et andmeid politseile edasi ei anta . Tulemusi saadakse siiski enam usaldusväärseid ainult mitteohtlike kuritegude suhtes. Keegi ei tunnista, et on tapnud, vägistanud. Tunnistatakse löömisi, väiksemaid vargusi jne. self-report näitab, kui palju on väiksemaid rikkumisi. Probleemiks ka see, et võib esineda vale-positiivseid – isik räägib, et on, aga tegelikult ei ole. Kolledži õpilaste seas võib olla selliseid, kes leiavad, et prestiižikas on enda kohta kirjutada

    5.2.2. Kannatanute uuringud
    Kajastab seda osa latentsusest, mis on kannatanule teada. Küsitletakse elanikkonda, see ei nõua nii suurt usaldust. Samas pika perioodi kohta pole mõtet küsida – unustatakse. Lühike periood on aga kulukas .
    Probleemid: osaline peegeldamine.
    Eestis on teatamine 33-50%. Kõige vähem latentsed on rasked isikuvastased kuriteod, mis ei ole seksuaalkuriteod. Paranenud on teatamine autovargustest.
    • Kannatanute uuringud – need, mis kannatanu teadmata ja kus kannatanut ei ole, jäävad välja. Küsitletakse elanikkonda – siin pole vastamine nii keeruline ja nii tohutut usaldust ei eelda . Saadakse paremaid tulemusi. Mitu korda on kannatanud teatud perioodi jooksul olnud kuritegude ohvriks. Kui pärida kogu elu koha, siis läheb arvestus väga keeruliseks. Probleemid: ei peegelda neid, kus ohvrit ei ole või ohver ei tea. Ei saa toetuda tegeliku kuritegevuse seisundi hindamisel ainult registreeritud ja latentsele läbi kannatanute uuringute. Probleemiks ka see, et kui lähme küsitlema, siis ta peab vastama teatud perioodi kohta, on eksimust: on sündmusi, mis on olnud väljaspool seda perioodi, võib valesti mäletada, et need olid perioodi sees. Samuti midagi võis olla perioodi sees, aga ta ei mäleta neid. On võimalus, et läheb üldse meelest ära või paigutab ekslikult väljapoole aja piiri. Võib juhtuda ka, et on väljamõeldud situatsioonid. Praktiliselt kõik need, kus politseisse ei tahtnud pöörduda, siis need probleemid teevad ka kannatanute uuringute tulemusi mitte kõiki sündmusi haaravaks. See, et kannatanu karda politseid, on väiksem takistus küsitlusele vastamiseks. Kannatanuuuringu puhul jääb teatav osa uuringut välja – nt illegaalsed immigrandid kipuvad välja jääma. Kui kardab kättemaksu, siis samuti pole välistatud, et ei räägi.
    Vahendid vigade vähendamiseks – lühem periood, mis vähendab mäletamisest tingitud vigu. Mida lühema perioodi kohta me küsime, seda kallimaks see uuring läheb. Tegelikult tahetase vastuseid inimpäeva kohta.
    Paljud isikud satuvad olukorda, kus nad on kuriteoohvrid olnud, aga ei teata, siis need tulemused on olnud omajagu kõikuvad, jäävad poole ja kolmandiku vahele – st teatatud saab 1/3 kuni ½ kuritegevusest. Teiste riikidega võrreldes on see üpris rutiinne. Peab arvestama et registreerimata on vähemalt sama palju kui registreeritud.
    Erinevate kuriteoliikude puhul on ohvrite poolt teatamise tõenäosus erinev. Kõige vähem latentsed on rasked isikuvastased kuriteod, mis ei ole seksuaalkuriteod. Kodudesse sissemurdmise teatamise % on ligi 50% ainult. Nt kopliliinidel on maju, kus inimesed jätavad meelega ukse luku panemata, sest kui lukus siis lõhutakse kindlalt ära. Üks kuriteoliik, millest teatamine on paranenud, on autovargused – 86% juures. Tekib küsimus, miks see 100% ei ole – autosid on erinevaid – on ka neid, mis kuigi palju ei maksa. Või on omanikustaatus ebaselge , pole dokumente vms.
    Püütakse saada hinnangut politseitegevusele – meil on uuringud näidanud politseile antava hinnangu tõusu – 2004 oli 32% kes arvas, et politseid käitub vastavalt nõuetele. Hinnang kuriteoohvrite poolelt – ootused politseitegevusele on muutunud realistlikumaks – kui 90-lõpus olid lootused liiga naiivsed, et kõik saab avastatud jne. politsei on ka ise muutunud palju professionaalsemaks.
    Uuritakse ka kui suur on elanikkonna hirm kuritegevuse ees – siin osa tulemusi on, et hirm on suurem neil, kellel on ohvriks langemise tõenäosus palju väiksem. Hirmu osas on mõnevõrra võimalik näha vähenemist – kui olete öösel oma elukohas väljaspool kodu, kas tunnete hirmu või mitte. See hirm on vähenenud – 1993 tundis end julgelt 51%, 2004 aga 68%.

    5.2.3. Muud meetodid latentse kuritegvuse uurimiseks
    Vaatlus – grupis või kõrvalvaatlus (nt liiklusväärteod). Teatud situatsiooni jaoks on self-report ainuke – nt tahetakse mingi kindla isiku kohta.
    On veel vähemlevinud latentse kuritegevuse uurimise meetodid – vaatlus – grupi seest (isik läheb teatud grupeeringu sisse ja peab päevikut selle kohta, millega vastav grupp hakkama sai ja pärast võrrelda, mis on sellest ka registris). Kõrvalt vaatlus (ennekõike väärtegude puhul – nt automaatne kiirusemõõtja kuskil teel. Liiklusvoo kiirused nt. tulemus on, et enamus piirkiirusest kinni ei pea. Vaatluseks võib pidada ka joobesjuhtimise kontrolli – puhuvad kõik!) Arvestuslik meetod – mingite muude andmete järgi saab teha järeldusi selle kohta, kas teatud kuritegusid pannakse vähem või rohkem toime – nr kui alkoholi kättesaadavus tugevasti langes ja siis samal ajal suhkru läbimüük kasvas. Ilmselt suhkur läks selleks, et illegaalne alkoholitootmine kasvaks. Korrelatsioonis ilusti. Kindlustusfirmad on ka uurinud kuritegeliku kindlustuspettuse tõenäosust – on võrreldud isikuid, kes on reisinud äriasjas ja neid, kes oma lõbuks ja vaadatud, kui palju on kindlustusjuhtumeid, kus on tahetud hüvitust – ärireisijatega on neid juhtumeid palju vähem kui oma lõbuks. Seega viimaste seas on põhjendamatut nõudmist märksa rohkem.
    5.3. Sotsioloogilised meetodid kriminoloogias
    5.3.1. Statistilise töötluse võimalused erinevate tunnuste
    tüüpide puhul
    5.3.2. Korrelatsioonianalüüs
    5.3.3. Regressioonianalüüs
    5.3.4. Sotsioloogiliste uuringute tulemuste usalduspiirid
    Kuritegevuse seisund (1 tund, 8. nädal)
    6.1. Kuritegvuse seisund Eestis
    6.2. Kuritegevuse seisund teistes riikides
    Kuritegevuse preventsioon
    Preventsioon jaguneb:
  • Üldpreventsioon (sotsiaalne, olustikuline e situatsiooniline)
  • Eripreventsioon
    Sotsiaalsed ennetusmeetmed – süütegusid põhjustavate tegurite mõjutamine sotsiaal-, haridus-, pere-, noorte-, kultuuri-, kiriku-, majandus-, liiklus - jms poliitika abil eesmärgiga luua eeldused iga inimese kaasamiseks ühiskonnaellu, et sel moel ära hoida tema hälbiv käitumine.
    Olustikulised ennetusmeetmed – süütegude toimepanemisele kalduvate inimeste või kriminogeensete olukordade ja kohtade mõjutamine süütegude ennetamise eesmärgil.
    Selle eesmärgi saavutamiseks tuleb:
  • raskendada süütegude toimepanemist
  • suurendada süütegude avastamise tõenäosust
  • vähendada süütegudega saadavat kasu
    Tapjatest on 73% üle 25 a, 15% vanuses 21-25, 6% vanuses 19-20, 7% kuni 18a.
    Kuritegusid 100 000 elaniku kohta vs tapmisi 100 000 elaniku kohta: suhteliselt võrdeline seos on nende vahel, kuid viimasel ajal on hakanud vahe pisut suurenema (tapmiseid vähem).
    T = 1 – SUMMA 4k/n (n-1)
    Preventsioonivahendeid liigitatakse mitmel erineval alusel – esmane, teisene, kolmas –
    Esmane – suunatud elanikkonnale tervikuna
    Teisene – suunatud riskirühmadele
    Kolmandane – konkreetsele isikule, kes juba on kuritegusid toime pannud.
    Negatiivne üldpreventsioon – näitab, et seadused on olemas ja pannakse tööle vastavate ametnike poolt. Siin ennekõike läbi hirmu, kui hirmus on karistus ja palju negatiivseid emotsioone see karistus indiviidis tekitab.
    Positiivne üldpreventsioon – pole niivõrd hirmutamine kuivõrd motivatsiooni tekitamine ühiskonnaliikmetes normipäraselt käituda. See peaks tekitama inimeses arusaama, et on vale kuidagimoodi käituda. Normide järgimine n mõistlik ja normipäraselt eesmärgi saavutamine on tõenäosem ja kui mitte järgida, siis mingeid eeliseid sellest ei saa.
    Eripreventsiooni toimemehhanismid – karistatava poolt järgmist kuritegu ennetada. Hirmutamine: inimene teist korda vangi sattuda ei taha. Ümberkasvatamise suund – nõustamine jne – sellega siiski väga palju tänapäeval ei saavuta. Eluaegne vangistus – isoleerimine , kuritegude toimepanemine väljaspool vanglat on lihtsalt võimatu. See on ka lühemaajalistel vangistustel. Seega teatud ajaks muudame inimese ohutuks.
    Esmane, teisene, kolmandane on
  • Vasakule Paremale
    Kriminoloogia konspekt #1 Kriminoloogia konspekt #2 Kriminoloogia konspekt #3 Kriminoloogia konspekt #4 Kriminoloogia konspekt #5 Kriminoloogia konspekt #6 Kriminoloogia konspekt #7 Kriminoloogia konspekt #8 Kriminoloogia konspekt #9 Kriminoloogia konspekt #10 Kriminoloogia konspekt #11 Kriminoloogia konspekt #12 Kriminoloogia konspekt #13 Kriminoloogia konspekt #14 Kriminoloogia konspekt #15 Kriminoloogia konspekt #16 Kriminoloogia konspekt #17 Kriminoloogia konspekt #18 Kriminoloogia konspekt #19 Kriminoloogia konspekt #20 Kriminoloogia konspekt #21 Kriminoloogia konspekt #22 Kriminoloogia konspekt #23 Kriminoloogia konspekt #24 Kriminoloogia konspekt #25 Kriminoloogia konspekt #26 Kriminoloogia konspekt #27 Kriminoloogia konspekt #28 Kriminoloogia konspekt #29 Kriminoloogia konspekt #30 Kriminoloogia konspekt #31 Kriminoloogia konspekt #32 Kriminoloogia konspekt #33 Kriminoloogia konspekt #34 Kriminoloogia konspekt #35 Kriminoloogia konspekt #36 Kriminoloogia konspekt #37 Kriminoloogia konspekt #38 Kriminoloogia konspekt #39 Kriminoloogia konspekt #40 Kriminoloogia konspekt #41 Kriminoloogia konspekt #42 Kriminoloogia konspekt #43 Kriminoloogia konspekt #44 Kriminoloogia konspekt #45 Kriminoloogia konspekt #46 Kriminoloogia konspekt #47 Kriminoloogia konspekt #48 Kriminoloogia konspekt #49 Kriminoloogia konspekt #50 Kriminoloogia konspekt #51 Kriminoloogia konspekt #52 Kriminoloogia konspekt #53 Kriminoloogia konspekt #54 Kriminoloogia konspekt #55 Kriminoloogia konspekt #56 Kriminoloogia konspekt #57 Kriminoloogia konspekt #58 Kriminoloogia konspekt #59 Kriminoloogia konspekt #60 Kriminoloogia konspekt #61 Kriminoloogia konspekt #62 Kriminoloogia konspekt #63 Kriminoloogia konspekt #64 Kriminoloogia konspekt #65 Kriminoloogia konspekt #66
    Punktid 50 punkti Autor soovib selle materjali allalaadimise eest saada 50 punkti.
    Leheküljed ~ 66 lehte Lehekülgede arv dokumendis
    Aeg2014-03-13 Kuupäev, millal dokument üles laeti
    Allalaadimisi 109 laadimist Kokku alla laetud
    Kommentaarid 1 arvamus Teiste kasutajate poolt lisatud kommentaarid
    Autor onuzim Õppematerjali autor
    KRIMINOLOOGIA TÄISKONSPEKT

    Sarnased õppematerjalid

    thumbnail
    25
    doc

    Kriminoloogia loeng

    Kriminoloogia ­ Jaan Ginter 2012 Registreeritud kuritegevus ei peegelda tegelikku olukorda, päriselt on see kuritegevus 2 korda suurem kui registreeritud kuritegevus. Seda seetõttu, et paljudel juhtudel puudub kannatanu või teisel juhul ei teata kannatanud kuriteost politseile. Registreeritud kuritegevuse numbril pole absoluutset tähendust, kuid teatav seos on ka latentsel kuritegevusel. Registreerimise kvaliteet võib palju muutuda, nt registratsioonisüsteemi muutuste tõttu. 1987 a. toimusid ka tegelikus elupildis palju muutusi, mis tolle aja registreeritud kuritegevuse numbrit muutsid. ,,Me ei saa valikuid tehes valida ainult häid asju" teatud head asjad võivad kaasa tuua ka negatiivseid tulemusi. Kuritegevuse vastu võitlemine on rahva heaolu mõttes küll oluline, aga see pole prioriteet. ´92-96 aastaid mõjutasid suurte radikaalsete muutuste lõppemine, kuid teisalt ühiskonna terviku tasemel seostati kuritegevuse taset sellega kuidas indiviidid, eesmärgid nin

    Kriminoloogia
    thumbnail
    34
    pdf

    KRIMINOLOOGIA EKSAMIKS

    piisavalt toetust, luuakse uus seadus, mille tulemusel opositsioonis oleva grupi huvid kannatavad. Kuritegevus, ideoloogia ja reaalsus Richard Quinney vaated on esindatud tema 1970.a. ilmunud raamatus ​The Social Reality of Crime​. Reaalsus on ideoloogiliselt konstrueeritud ehkvõimugruppidel on võimalus defineerida reaalsust nii, et see on neile kasulik. Sel moel kuritegu ei ole midagi, mis on “kurjus” oma loomu poolest, vaid sotsiaalne konstruktsioon võimulolijate kasuks. Uus kriminoloogia }​ Tuntud radikaalse kriminoloogia esitluseks on 1973. a ilmunud ​Ian Taylori, Paul Waltoni ja Jock Youngi​The New Criminology.​ }​ Kuritegevust tuleb vaadelda protsessina, millel on eri tasandeid, kuid milles ühtki tegurit ei saa unustada või tervikust lahti kiskuda. Steven Spitzeri teooria }​ Spitzer oletas, et kapitalistliku ühiskonna aluseks on klassikonflikt, harmooniat saavutatakse ühe spetsiifilise klassi domineerimisega. }​

    Kriminoloogia
    thumbnail
    33
    doc

    Kriminaalpoliitika kordamine

    Kriminaalseadus seletab ja põhjustab selle valdkonna. Miks seda võetakse vastu sellisena? H. Göpperger: kriminaalõiguse reform. Kriminaalpoliitika on teadus kriminaalõiguse reformimisest. Kriminaalõiguse reformi dogmaatiline põhjendamine- karistamine on kuriteo eitamine. Kannatanu afekt- peab leidma rahu, kui riik karistab kuritegejat kuritegevuse eest ning see peab olema range. 3) seob kriminaalpoliitika kriminoloogiaga. Need on väga sarnased teadused. Kriminoloogia - teadus kuritegevusest, on empiiriline teadus, mis püüab olla õiguspoliitiliselt neutraalne. Garapalo 1885. a kasutas esimest korda kriminoloogia mõistet. Lobmraso 1876. a - modernse kriminoloogia lähenemine. Kriminaalpoliitika on väärtusteadus, ei ole neutraalne. See ongi vahe kriminoloogia ja kriminaalpoliitika vahel. Kriminoloogia arusaam - kui komplektne ja üldistatud teadmiste kogum kuritegevusest, mis tuleneb kriminoloogia teadusest. Teadmiste kogum koosneb 2 poolest:

    Õigusteadus
    thumbnail
    17
    doc

    Hälbiva käitumise sotsioloogia (TLÜ) konspekt 2012

    toime naised. - Kõige väiksem on naiste osakaal üldohtlike (nt süütamine), alaealiste vastu suunatud ja seksuaalkuritegudes. - Varavastaste kuritegude sooritamise puhul on naiste ja meeste suhe 1:9. - Ainult naine saab sooritada üht teatud kuritegu: KarS § 116. Lapse tapmine ­ Ema poolt oma sündiva või vastsündinud lapse tapmise eest ­ karistatakse kuni viieaastase vangistusega. Feministlik kriminoloogia - Bioloogilised ja psühholoogilised seletused Põhjused naise bioloogilistes ja psühholoogilistes omadustes Naise loomus, pärilikud bioloogilised omadused - Passiivsem loomus - Tegevust suunab emainstinkt - Sotsioloogilised seletused Põhjused naise sotsiaalses rollis - Kuuletuv, passiivne, sõltuv, kohanev - Alalhoidlik - Ühiskonna osalemisvaldkondade ahtrus (nappus) 1980ndate uurimissuunad

    Hälbiva käitumise sotsioloogia
    thumbnail
    7
    doc

    Kriminoloogia ajalooline areng

    Õigusteaduskond Kriminoloogia ajalooline areng Essee Kriminoloogia TALLINN 2008 2 Sissejuhatus Ajalooga on üks kummaline asi, ajalugu oli ammu enne meid, ent samas on ajalugu ka praegune hetk. Iga päev toimub maailmas midagi murrangulist, midagi, mis on homseks juba harilik ning tavapärane ja ehk unustatudki. Keegi tark on kunagi öelnud: ,,Pole olemas midagi uut siin ilmas, on ainult unustatud vana." Sellega tuleb ilmselt nõustuda.

    Õigus
    thumbnail
    73
    doc

    Kuritegude viktimoloogiline analüüs kui preventsiooniline koosseis

    ..................................................................................................74 3 SISSEJUHATUS Kuriteoennetuse viktimoloogiline suund on erakordselt perspektiivne kuritegevuse vastu võitlemise alaliik, võttes arvesse, et kuritegevus kujutab endast peamisi ühiskonda, selle demokraatlikku ja majanduslikku arengut ohustavaid tegureid XXI sajandil. Käesoleval ajal, kui kriminoloogia käsutuses on olemas vajalikud materjalid kurjategija isiku ja tema käitumise kohta, säilib endiselt vajadus andmetes nende kohta, kes on langenud vägivalla või varguse ohvriks. Andmed nende isikute kohta, nende isikuomaduste analüüs, taoliste andmete üldistamine koos kurjategija isikuomaduste uurimisega saab aidata paremini määrata profülaktiliste meetmete suunda, eristada inimrühmi, kes enim sageli satuvad ühe või teise ühiskondlikult ohtliku ründe alla, s

    Kriminoloogia
    thumbnail
    11
    doc

    Kuritegevus Eestis

    LÄÄNE-VIRU RAKENDUSKÕRGKOOL Sotsiaaltöö õppetool I ST Annika Kukke KURITEGEVUS Referaat Õppejõud: Reino Veielainen Mõdriku 2010 SISUKORD LÄÄNE-ViRU RAKENDUSKÕRGKOOL.......................................................................1 sisukord............................................................................................................................... 2 SISSEJUHATUS................................................................................................................ 3 0.1 Tapmine..................................................................................................................... 4 0.2 Varavastased kuriteod................................................................................................4 0.3 Organiseeritud kuritegevus......

    Sotsioloogia
    thumbnail
    15
    doc

    Kuritegevus Eestis

    Kuritegevus Eestis 20. saj lõpul ja 21.saj algul Uurimistöö Nimi: Kool: Sisukord 1. Sissejuhatus..............................................................................................3 2. Kuritegevus..............................................................................................4 3. Võrdlus teiste riikidega............................................................................5 4. Kuritegevus Eestis..................................................................................6 - Kuritegude avastamine..................................................................6 - Erinevat liiki kuritegude ohvriks langemine..................................7 5. Röövimised ja isiklike esemete vargused.................................................9 6. Tapmis

    Uurimistöö




    Kommentaarid (1)

    lisetr profiilipilt
    02:22 16-01-2020



    Sellel veebilehel kasutatakse küpsiseid. Kasutamist jätkates nõustute küpsiste ja veebilehe üldtingimustega Nõustun